Sei sulla pagina 1di 38

"sections":{"Physics":{"subsections":[{"id":"45863","paper_id":"12803","section":"Physics","subsection":"SEC-

A","sequence_id":"1","sequence_sid":"0","header":"<ul>\r\n\t<li>This section contains <b>TWENTY<\/b>


questions.<\/li>\r\n\t<li>Each question has FOUR options (A), (B), (C) and (D). <b>ONLY ONE<\/b> of these four
options is correct.<\/li>\r\n\t<li>For each question, select the alphabet corresponding to the correct option
provided below the questions.<\/li>\r\n\t<li><b>Marking scheme:<\/b>&nbsp;\r\n\t<ul>\r\n\t\t<li>Full Marks :
<span style=\"color:black\">+4 If correct answer is selected.<\/span><\/li>\r\n\t\t<li><span
style=\"color:black\">Zero Marks <\/span>: <span style=\"color:black\">0&nbsp;If none of the option is
selected.<\/span><\/li>\r\n\t\t<li><span style=\"color:black\">Negative Marks <\/span>: <span
style=\"color:black\">&ndash;1 If wrong option is
selected.<\/span><\/li>\r\n\t<\/ul>\r\n\t<\/li>\r\n<\/ul>\r\n","num_grps":"20","marks":"80","neg_marks":"0","ti
me_duration":"40","type":"paper","ans_limit":"0","gdata":[{"id":"911216","section_id":"45863","sequence_id":"1
","sequence_oid":null,"render_gid":"","num_ques":"1","header":null,"content":null,"exam_id":"80","topic_id":"0"
,"is_linked":null,"autoHeader":"","qdata":{"1":{"subquestion":[{"id":"993292","group_id":"911216","sequence_id"
:"1","sequence_oid":null,"sequence_sid":"0",

"render_qid":"1","render_sqid":"","header":null,"type":"objective","category":"none","is_mco":"no","level":"low"
,"time":"120","marks":"4","neg_marks":"1","meta_id":"3353","dirty":null,"comment":null,"history":"[<b>Feb 05
2021 12:03:06][admin<\/b>][I][<b>admin<\/b>]","answer":"4","prefix":null,"content":"<div style=\"text-
align:justify\">The frequency of a tunning fork is 384 per second and velocity of sound in air is 352 m\/s. How far
the sound has traversed while fork completes 36 vibration :-
<\/div>\n","suffix":null,"testcases":[],"solution":null,"insert_time":"2021-02-05
12:03:06","login_id":"admin","topic_id":0,"hint":null,"qmock_list":null,"unattempt":"0","unattempt_avg_time":"4
5","unevaluate":"0","unevaluate_avg_time":"45","correct":"0","correct_avg_time":"45","incorrect":"2","incorrect
_avg_time":"376.33","sp_answer":"","mco_marks":"0","exam_ques":"no","exam_info":null,"sub_type":null,"is_bo
nus":"0","is_extra_question":"no","answer_type":null,"integer_length":null,"error_status":"0","source_q_id":"146
6605","topic":"miscellaneous","subject":"physics","options":["3m&nbsp;","13 m&nbsp;","23 m &nbsp;","33
m"]}]}}},{"id":"911217","section_id":"45863","sequence_id":"2","sequence_oid":null,"render_gid":"","num_ques":
"1","header":null,"content":null,"exam_id":"80","topic_id":"0","is_linked":null,"autoHeader":"","qdata":{"2":{"sub
question":[{"id":"993293","group_id":"911217","sequence_id":"2","sequence_oid":null,"sequence_sid":"0",

"render_qid":"2","render_sqid":"","header":null,"type":"objective","category":"none","is_mco":"no","level":"low"
,"time":"120","marks":"4","neg_marks":"1","meta_id":"3353","dirty":null,"comment":null,"history":"[<b>Feb 05
2021 12:03:06][admin<\/b>][I][<b>admin<\/b>]","answer":"4","prefix":null,"content":"If the air pressure is
doubled at constant temperature, then the speed of sound will be :-
","suffix":null,"testcases":[],"solution":null,"insert_time":"2021-02-05
12:03:06","login_id":"admin","topic_id":0,"hint":null,"qmock_list":null,"unattempt":"2","unattempt_avg_time":"1
6.33","unevaluate":"0","unevaluate_avg_time":"45","correct":"0","correct_avg_time":"45","incorrect":"0","incorr
ect_avg_time":"45","sp_answer":"","mco_marks":"0","exam_ques":"no","exam_info":null,"sub_type":null,"is_bon
us":"0","is_extra_question":"no","answer_type":null,"integer_length":null,"error_status":"0","source_q_id":"1466
613","topic":"miscellaneous","subject":"physics","options":["Double","Three times","Four times","Equal to its
initial
value"]}]}}},{"id":"911218","section_id":"45863","sequence_id":"3","sequence_oid":null,"render_gid":"","num_qu
es":"1","header":null,"content":null,"exam_id":"80","topic_id":"0","is_linked":null,"autoHeader":"","qdata":{"3":{"
subquestion":[{"id":"993294","group_id":"911218","sequence_id":"3","sequence_oid":null,"sequence_sid":"0","
render_qid":"3","render_sqid":"","header":null,"type":"objective","category":"none","is_mco":"no","level":"low",
"time":"120","marks":"4","neg_marks":"1","meta_id":"3353","dirty":null,"comment":null,"history":"[<b>Feb 05
2021 12:03:06][admin<\/b>][I][<b>admin<\/b>]","answer":"2","prefix":null,"content":"At what temperature the
speed of sound in air becomes double of its value at 0&ordm;C :-
","suffix":null,"testcases":[],"solution":null,"insert_time":"2021-02-05
12:03:06","login_id":"admin","topic_id":0,"hint":null,"qmock_list":null,"unattempt":"2","unattempt_avg_time":"1
6.67","unevaluate":"0","unevaluate_avg_time":"45","correct":"0","correct_avg_time":"45","incorrect":"0","incorr
ect_avg_time":"45","sp_answer":"","mco_marks":"0","exam_ques":"no","exam_info":null,"sub_type":null,"is_bon
us":"0","is_extra_question":"no","answer_type":null,"integer_length":null,"error_status":"0","source_q_id":"1466
617","topic":"miscellaneous","subject":"physics","options":["273 &ordm;C","819 &ordm;C","1092 &ordm;C","553
&ordm;C"]}]}}},{"id":"911219","section_id":"45863","sequence_id":"4","sequence_oid":null,"render_gid":"","num
_ques":"1","header":null,"content":null,"exam_id":"80","topic_id":"0","is_linked":null,"autoHeader":"","qdata":{"
4":{"subquestion":[{"id":"993295","group_id":"911219","sequence_id":"4","sequence_oid":null,"sequence_sid":"0
",

"render_qid":"4","render_sqid":"","header":null,"type":"objective","category":"none","is_mco":"no","level":"medi
um","time":"120","marks":"4","neg_marks":"1","meta_id":"3353","dirty":null,"comment":null,"history":"[<b>Feb
05 2021 12:03:06][admin<\/b>][I][<b>admin<\/b>]","answer":"1","prefix":null,"content":"<div style=\"text-
align:justify\">An open pipe is suddenly closed at one end with the result that the frequency of third harmonic of
the closed pipe is found to be higher by 100 Hz than the fundamental frequency of the open pipe. The
fundamental frequency of the open pipe
is:&ndash;<\/div>\n","suffix":null,"testcases":[],"solution":null,"insert_time":"2021-02-05
12:03:06","login_id":"admin","topic_id":0,"hint":null,"qmock_list":null,"unattempt":"2","unattempt_avg_time":"2
1.33","unevaluate":"0","unevaluate_avg_time":"45","correct":"0","correct_avg_time":"45","incorrect":"0","incorr
ect_avg_time":"45","sp_answer":"","mco_marks":"0","exam_ques":"no","exam_info":null,"sub_type":null,"is_bon
us":"0","is_extra_question":"no","answer_type":null,"integer_length":null,"error_status":"0","source_q_id":"1466
620","topic":"miscellaneous","subject":"physics","options":["200 Hz","300 Hz","240 Hz","480
Hz"]}]}}},{"id":"911220","section_id":"45863","sequence_id":"5","sequence_oid":null,"render_gid":"","num_ques"
:"1","header":null,"content":null,"exam_id":"80","topic_id":"0","is_linked":null,"autoHeader":"","qdata":{"5":{"su
bquestion":[{"id":"993296","group_id":"911220","sequence_id":"5","sequence_oid":null,"sequence_sid":"0"

,"render_qid":"5","render_sqid":"","header":null,"type":"objective","category":"none","is_mco":"no","level":"low
","time":"120","marks":"4","neg_marks":"1","meta_id":"3353","dirty":null,"comment":null,"history":"[<b>Feb 05
2021 12:03:07][admin<\/b>][I][<b>admin<\/b>]","answer":"3","prefix":null,"content":"<div style=\"text-
align:justify\">An organ pipe P closed at one end vibrates in its first harmonic. Another organ pipe Q open at both
ends vibrates in its third harmonic. When both are in resonance with a tuning fork, the ratio of the length of P to
that of Q is :-<\/div>\n","suffix":null,"testcases":[],"solution":null,"insert_time":"2021-02-05
12:03:07","login_id":"admin","topic_id":0,"hint":null,"qmock_list":null,"unattempt":"2","unattempt_avg_time":"1
7.33","unevaluate":"0","unevaluate_avg_time":"45","correct":"0","correct_avg_time":"45","incorrect":"0","incorr
ect_avg_time":"45","sp_answer":"","mco_marks":"0","exam_ques":"no","exam_info":null,"sub_type":null,"is_bon
us":"0","is_extra_question":"no","answer_type":null,"integer_length":null,"error_status":"0","source_q_id":"1466
630","topic":"miscellaneous","subject":"physics","options":["<img alt=\"frac{1}{2}\"
src=\"https:\/\/d2lbh14zkcqlst.cloudfront.net\/content_data\/content_images\/16122634101979691940.png\"
\/>","<img alt=\"frac{1}{4}\"
src=\"https:\/\/d2lbh14zkcqlst.cloudfront.net\/content_data\/content_images\/16122634161943618234.png\"
\/>","<img alt=\"frac{1}{6}\"
src=\"https:\/\/d2lbh14zkcqlst.cloudfront.net\/content_data\/content_images\/16122634221723531574.png\"
\/>","<img alt=\"frac{1}{8}\"
src=\"https:\/\/d2lbh14zkcqlst.cloudfront.net\/content_data\/content_images\/1612263427498035955.png\"
\/>"]}]}}},{"id":"911221","section_id":"45863","sequence_id":"6","sequence_oid":null,"render_gid":"","num_ques"
:"1","header":null,"content":null,"exam_id":"80","topic_id":"0","is_linked":null,"autoHeader":"","qdata":{"6":{"su
bquestion":[{"id":"993297","group_id":"911221","sequence_id":"6","sequence_oid":null,"sequence_sid":"0",

"render_qid":"6","render_sqid":"","header":null,"type":"objective","category":"none","is_mco":"no","level":"medi
um","time":"120","marks":"4","neg_marks":"1","meta_id":"3353","dirty":null,"comment":null,"history":"[<b>Feb
05 2021 12:03:07][admin<\/b>][I][<b>admin<\/b>]","answer":"1","prefix":null,"content":"Two open organ pipes of
length 25 cm and 25.5 cm produce 10 beat\/sec. The velocity of sound will be :-
","suffix":null,"testcases":[],"solution":null,"insert_time":"2021-02-05
12:03:07","login_id":"admin","topic_id":0,"hint":null,"qmock_list":null,"unattempt":"2","unattempt_avg_time":"1
7.33","unevaluate":"0","unevaluate_avg_time":"45","correct":"0","correct_avg_time":"45","incorrect":"0","incorr
ect_avg_time":"45","sp_answer":"","mco_marks":"0","exam_ques":"no","exam_info":null,"sub_type":null,"is_bon
us":"0","is_extra_question":"no","answer_type":null,"integer_length":null,"error_status":"0","source_q_id":"1466
636","topic":"miscellaneous","subject":"physics","options":["255 m\/s","250 m\/s","350 m\/s","None of
these"]}]}}},{"id":"911222","section_id":"45863","sequence_id":"7","sequence_oid":null,"render_gid":"","num_qu
es":"1","header":null,"content":null,"exam_id":"80","topic_id":"0","is_linked":null,"autoHeader":"","qdata":{"7":{"
subquestion":[{"id":"993298","group_id":"911222","sequence_id":"7","sequence_oid":null,"sequence_sid":"0

","render_qid":"7","render_sqid":"","header":null,"type":"objective","category":"none","is_mco":"no","level":"lo
w","time":"120","marks":"4","neg_marks":"1","meta_id":"3353","dirty":null,"comment":null,"history":"[<b>Feb
05 2021 12:03:07][admin<\/b>][I][<b>admin<\/b>]","answer":"3","prefix":null,"content":"<div style=\"text-
align:justify\">Frequency of tuning fork A is 256 Hz. It produces 4 beats\/second with tuning fork B. When wax is
applied at tuning fork B then 6 beats\/second are heard. Frequency of B
is:&ndash;<\/div>\n","suffix":null,"testcases":[],"solution":null,"insert_time":"2021-02-05
12:03:07","login_id":"admin","topic_id":0,"hint":null,"qmock_list":null,"unattempt":"2","unattempt_avg_time":"1
7.33","unevaluate":"0","unevaluate_avg_time":"45","correct":"0","correct_avg_time":"45","incorrect":"0","incorr
ect_avg_time":"45","sp_answer":"","mco_marks":"0","exam_ques":"no","exam_info":null,"sub_type":null,"is_bon
us":"0","is_extra_question":"no","answer_type":null,"integer_length":null,"error_status":"0","source_q_id":"1466
642","topic":"miscellaneous","subject":"physics","options":["250 Hz","260 Hz","252 Hz &nbsp;","None of
these"]}]}}},{"id":"911223","section_id":"45863","sequence_id":"8","sequence_oid":null,"render_gid":"","num_qu
es":"1","header":null,"content":null,"exam_id":"80","topic_id":"0","is_linked":null,"autoHeader":"","qdata":{"8":{"
subquestion":[{"id":"993299","group_id":"911223","sequence_id":"8","sequence_oid":null,"sequence_sid":"0",

"render_qid":"8","render_sqid":"","header":null,"type":"objective","category":"none","is_mco":"no","level":"high
","time":"120","marks":"4","neg_marks":"1","meta_id":"3353","dirty":null,"comment":null,"history":"[<b>Feb 05
2021 12:03:07][admin<\/b>][I][<b>admin<\/b>]","answer":"2","prefix":null,"content":"<div style=\"text-
align:justify\">A whistle revolves in a circle with angular speed &omega; = 20 rad\/s using a string of length 50 cm.
If the frequency of sound from the whistle is 385 Hz, then what is the minimum frequency heared by an observer
which is far away from the centre : (v<sub>sound<\/sub> = 340
m\/s)<\/div>\n","suffix":null,"testcases":[],"solution":null,"insert_time":"2021-02-05
12:03:07","login_id":"admin","topic_id":0,"hint":null,"qmock_list":null,"unattempt":"2","unattempt_avg_time":"1
8.33","unevaluate":"0","unevaluate_avg_time":"45","correct":"0","correct_avg_time":"45","incorrect":"0","incorr
ect_avg_time":"45","sp_answer":"","mco_marks":"0","exam_ques":"no","exam_info":null,"sub_type":null,"is_bon
us":"0","is_extra_question":"no","answer_type":null,"integer_length":null,"error_status":"0","source_q_id":"1466
655","topic":"miscellaneous","subject":"physics","options":["385 Hz","374 Hz","394 Hz","333
Hz"]}]}}},{"id":"911224","section_id":"45863","sequence_id":"9","sequence_oid":null,"render_gid":"","num_ques"
:"1","header":null,"content":null,"exam_id":"80","topic_id":"0","is_linked":null,"autoHeader":"","qdata":{"9":{"su
bquestion":[{"id":"993300","group_id":"911224","sequence_id":"9","sequence_oid":null,"sequence_sid":"0"

"render_qid":"9","render_sqid":"","header":null,"type":"objective","category":"none","is_mco":"no","level":"low"
,"time":"120","marks":"4","neg_marks":"1","meta_id":"3353","dirty":null,"comment":null,"history":"[<b>Feb 05
2021 12:03:07][admin<\/b>][I][<b>admin<\/b>]","answer":"4","prefix":null,"content":"<div style=\"text-
align:justify\">The rms velocity of a gas at a given temperature is 300 m\/s. What will be the rms velocity of a gas
having twice the molecular weight and half the temperature in K :-
<\/div>\n","suffix":null,"testcases":[],"solution":null,"insert_time":"2021-02-05
12:03:07","login_id":"admin","topic_id":0,"hint":null,"qmock_list":null,"unattempt":"2","unattempt_avg_time":"1
6.67","unevaluate":"0","unevaluate_avg_time":"45","correct":"0","correct_avg_time":"45","incorrect":"0","incorr
ect_avg_time":"45","sp_answer":"","mco_marks":"0","exam_ques":"no","exam_info":null,"sub_type":null,"is_bon
us":"0","is_extra_question":"no","answer_type":null,"integer_length":null,"error_status":"0","source_q_id":"1466
663","topic":"miscellaneous","subject":"physics","options":["300 m\/s","600 m\/s","75 m\/s","150
m\/s"]}]}}},{"id":"911225","section_id":"45863","sequence_id":"10","sequence_oid":null,"render_gid":"","num_qu
es":"1","header":null,"content":null,"exam_id":"80","topic_id":"0","is_linked":null,"autoHeader":"","qdata":{"10":
{"subquestion":[{"id":"993301","group_id":"911225","sequence_id":"10","sequence_oid":null,"sequence_sid":"0",

"render_qid":"10","render_sqid":"","header":null,"type":"objective","category":"none","is_mco":"no","level":"low
","time":"120","marks":"4","neg_marks":"1","meta_id":"3353","dirty":null,"comment":null,"history":"[<b>Feb 05
2021 12:03:09][admin<\/b>][I][<b>admin<\/b>]","answer":"4","prefix":null,"content":"Which of following is wrong
:-","suffix":null,"testcases":[],"solution":null,"insert_time":"2021-02-05
12:03:09","login_id":"admin","topic_id":0,"hint":null,"qmock_list":null,"unattempt":"2","unattempt_avg_time":"1
7","unevaluate":"0","unevaluate_avg_time":"45","correct":"0","correct_avg_time":"45","incorrect":"0","incorrect
_avg_time":"45","sp_answer":"","mco_marks":"0","exam_ques":"no","exam_info":null,"sub_type":null,"is_bonus"
:"0","is_extra_question":"no","answer_type":null,"integer_length":null,"error_status":"0","source_q_id":"146667
4","topic":"miscellaneous","subject":"physics","options":["<img alt=\"{C_V} = frac{R}{{gamma \u00a0- 1}}\"
src=\"https:\/\/d2lbh14zkcqlst.cloudfront.net\/content_data\/content_images\/1612263679347501850.png\"
\/>","<img alt=\"{C_P} = frac{{gamma R}}{{gamma \u00a0- 1}}\"
src=\"https:\/\/d2lbh14zkcqlst.cloudfront.net\/content_data\/content_images\/16122636851347372490.png\"
\/>","<img alt=\"frac{{{C_P}}}{{{C_V}}} = gamma \"
src=\"https:\/\/d2lbh14zkcqlst.cloudfront.net\/content_data\/content_images\/16122636901875175853.png\"
\/>","C<sub>P<\/sub> &ndash; C<sub>V<\/sub> =&nbsp;<img alt=\"frac{R}{2}\"
src=\"https:\/\/d2lbh14zkcqlst.cloudfront.net\/content_data\/content_images\/16122637001707009289.png\"
\/>"]}]}}},{"id":"911226","section_id":"45863","sequence_id":"11","sequence_oid":null,"render_gid":"","num_que
s":"1","header":null,"content":null,"exam_id":"80","topic_id":"0","is_linked":null,"autoHeader":"","qdata":{"11":{"
subquestion":[{"id":"993302","group_id":"911226","sequence_id":"11","sequence_oid":null,"sequence_sid":"0",

"render_qid":"11","render_sqid":"","header":null,"type":"objective","category":"none","is_mco":"no","level":"me
dium","time":"120","marks":"4","neg_marks":"1","meta_id":"3353","dirty":null,"comment":null,"history":"[<b>Fe
b 05 2021 12:03:09][admin<\/b>][I][<b>admin<\/b>]","answer":"3","prefix":null,"content":"<div style=\"text-
align:justify\">X is a closed vessel of volume V and contains O<sub>2<\/sub> at pressure P and temperature T. Y is
another closed vessel of same volume and it contains He at same temperature but at pressure 2P. Ratio of masses
of O<sub>2<\/sub> and He in vessels X and Y is :-
<\/div>\n","suffix":null,"testcases":[],"solution":null,"insert_time":"2021-02-05
12:03:09","login_id":"admin","topic_id":0,"hint":null,"qmock_list":null,"unattempt":"2","unattempt_avg_time":"1
7","unevaluate":"0","unevaluate_avg_time":"45","correct":"0","correct_avg_time":"45","incorrect":"0","incorrect
_avg_time":"45","sp_answer":"","mco_marks":"0","exam_ques":"no","exam_info":null,"sub_type":null,"is_bonus"
:"0","is_extra_question":"no","answer_type":null,"integer_length":null,"error_status":"0","source_q_id":"146668
0","topic":"miscellaneous","subject":"physics","options":["2 : 1","1 : 2","4 : 1","1 :
4"]}]}}},{"id":"911227","section_id":"45863","sequence_id":"12","sequence_oid":null,"render_gid":"","num_ques"
:"1","header":null,"content":null,"exam_id":"80","topic_id":"0","is_linked":null,"autoHeader":"","qdata":{"12":{"s
ubquestion":[{"id":"993303","group_id":"911227","sequence_id":"12","sequence_oid":null,"sequence_sid":"0",

"render_qid":"12","render_sqid":"","header":null,"type":"objective","category":"none","is_mco":"no","level":"low
","time":"120","marks":"4","neg_marks":"1","meta_id":"3353","dirty":null,"comment":null,"history":"[<b>Feb 05
2021 12:03:09][admin<\/b>][I][<b>admin<\/b>]","answer":"1","prefix":null,"content":"For V versus T curves at
&nbsp;constant pressure P<sub>1<\/sub> and P<sub>2<\/sub> for an ideal gas shown in Fig. :-\n<div style=\"text-
align:center\"><img
src=\"https:\/\/d2lbh14zkcqlst.cloudfront.net\/content_data\/content_images\/16122640532086755532phpOXo
BHlimage.png\" style=\"vertical-align: middle;height:137px; width:180px\"
\/><\/div>\n","suffix":null,"testcases":[],"solution":null,"insert_time":"2021-02-05
12:03:09","login_id":"admin","topic_id":0,"hint":null,"qmock_list":null,"unattempt":"2","unattempt_avg_time":"1
7","unevaluate":"0","unevaluate_avg_time":"45","correct":"0","correct_avg_time":"45","incorrect":"0","incorrect
_avg_time":"45","sp_answer":"","mco_marks":"0","exam_ques":"no","exam_info":null,"sub_type":null,"is_bonus"
:"0","is_extra_question":"no","answer_type":null,"integer_length":null,"error_status":"0","source_q_id":"146670
3","topic":"miscellaneous","subject":"physics","options":["P<sub>1<\/sub> &gt;
P<sub>2<\/sub>","P<sub>1<\/sub> &lt; P<sub>2<\/sub>","P<sub>1<\/sub> = P<sub>2<\/sub>","None of
these"]}]}}},{"id":"911228","section_id":"45863","sequence_id":"13","sequence_oid":null,"render_gid":"","num_q
ues":"1","header":null,"content":null,"exam_id":"80","topic_id":"0","is_linked":null,"autoHeader":"","qdata":{"13"
:{"subquestion":[{"id":"993304","group_id":"911228","sequence_id":"13","sequence_oid":null,"sequence_sid":"0"

"render_qid":"13","render_sqid":"","header":null,"type":"objective","category":"none","is_mco":"no","level":"low
","time":"120","marks":"4","neg_marks":"1","meta_id":"3353","dirty":null,"comment":null,"history":"[<b>Feb 05
2021 12:03:09][admin<\/b>][I][<b>admin<\/b>]","answer":"4","prefix":null,"content":"A monoatomic gas
(&gamma; = 5\/3) is suddenly compressed to (1\/8) its volume adiabatically; then the pressure of the gas will
change to :-&nbsp;","suffix":null,"testcases":[],"solution":null,"insert_time":"2021-02-05
12:03:09","login_id":"admin","topic_id":0,"hint":null,"qmock_list":null,"unattempt":"2","unattempt_avg_time":"1
8","unevaluate":"0","unevaluate_avg_time":"45","correct":"0","correct_avg_time":"45","incorrect":"0","incorrect
_avg_time":"45","sp_answer":"","mco_marks":"0","exam_ques":"no","exam_info":null,"sub_type":null,"is_bonus"
:"0","is_extra_question":"no","answer_type":null,"integer_length":null,"error_status":"0","source_q_id":"146670
8","topic":"miscellaneous","subject":"physics","options":["24\/5","8","40\/3&nbsp;","32&nbsp;"]}]}}},{"id":"91122
9","section_id":"45863","sequence_id":"14","sequence_oid":null,"render_gid":"","num_ques":"1","header":null,"c
ontent":null,"exam_id":"80","topic_id":"0","is_linked":null,"autoHeader":"","qdata":{"14":{"subquestion":[{"id":"9
93305","group_id":"911229","sequence_id":"14","sequence_oid":null,"sequence_sid":"0"
,"render_qid":"14","render_sqid":"","header":null,"type":"objective","category":"none","is_mco":"no","level":"lo
w","time":"120","marks":"4","neg_marks":"1","meta_id":"3353","dirty":null,"comment":null,"history":"[<b>Feb
05 2021 12:03:09][admin<\/b>][I][<b>admin<\/b>]","answer":"1","prefix":null,"content":"In an isobaric process of
diatomic gas internal energy increases by 400J, then work done by the gas is:-
","suffix":null,"testcases":[],"solution":null,"insert_time":"2021-02-05
12:03:09","login_id":"admin","topic_id":0,"hint":null,"qmock_list":null,"unattempt":"2","unattempt_avg_time":"1
7.33","unevaluate":"0","unevaluate_avg_time":"45","correct":"0","correct_avg_time":"45","incorrect":"0","incorr
ect_avg_time":"45","sp_answer":"","mco_marks":"0","exam_ques":"no","exam_info":null,"sub_type":null,"is_bon
us":"0","is_extra_question":"no","answer_type":null,"integer_length":null,"error_status":"0","source_q_id":"1466
709","topic":"miscellaneous","subject":"physics","options":["160J","&ndash;160J&nbsp;","200J&nbsp;","&ndash;2
00J"]}]}}},{"id":"911230","section_id":"45863","sequence_id":"15","sequence_oid":null,"render_gid":"","num_que
s":"1","header":null,"content":null,"exam_id":"80","topic_id":"0","is_linked":null,"autoHeader":"","qdata":{"15":{"
subquestion":[{"id":"993306","group_id":"911230","sequence_id":"15","sequence_oid":null,"sequence_sid":"0",

"render_qid":"15","render_sqid":"","header":null,"type":"objective","category":"none","is_mco":"no","level":"me
dium","time":"120","marks":"4","neg_marks":"1","meta_id":"3353","dirty":null,"comment":null,"history":"[<b>Fe
b 05 2021 12:03:10][admin<\/b>][I][<b>admin<\/b>]","answer":"3","prefix":null,"content":"<div style=\"text-
align:justify\">A prefect gas is found to obey the relation PV<sup>3\/2<\/sup> = constant during an adiabatic
process. If such a gas initially at a temperature T, is compressed to half of its initial volume, then its final
temperature will be:-<\/div>\n","suffix":null,"testcases":[],"solution":null,"insert_time":"2021-02-05
12:03:10","login_id":"admin","topic_id":0,"hint":null,"qmock_list":null,"unattempt":"2","unattempt_avg_time":"1
7","unevaluate":"0","unevaluate_avg_time":"45","correct":"0","correct_avg_time":"45","incorrect":"0","incorrect
_avg_time":"45","sp_answer":"","mco_marks":"0","exam_ques":"no","exam_info":null,"sub_type":null,"is_bonus"
:"0","is_extra_question":"no","answer_type":null,"integer_length":null,"error_status":"0","source_q_id":"146671
6","topic":"miscellaneous","subject":"physics","options":["2T","4T","(2)<sup>1\/2<\/sup>T","2<img alt=\"sqrt 2 \"
src=\"https:\/\/d2lbh14zkcqlst.cloudfront.net\/content_data\/content_images\/16122642481895311150.png\"
\/>&nbsp;T"]}]}}},{"id":"911231","section_id":"45863","sequence_id":"16","sequence_oid":null,"render_gid":"","n
um_ques":"1","header":null,"content":null,"exam_id":"80","topic_id":"0","is_linked":null,"autoHeader":"","qdata"
:{"16":{"subquestion":[{"id":"993307","group_id":"911231","sequence_id":"16","sequence_oid":null,"sequence_si
d":"0"

,"render_qid":"16","render_sqid":"","header":null,"type":"objective","category":"none","is_mco":"no","level":"lo
w","time":"120","marks":"4","neg_marks":"1","meta_id":"3353","dirty":null,"comment":null,"history":"[<b>Feb
05 2021 12:03:10][admin<\/b>][I][<b>admin<\/b>]","answer":"3","prefix":null,"content":"<div style=\"text-
align:justify\">A Carnot engine has efficiency 1\/5. Efficiency becomes 1\/3 when temperature of sink is decreased
by 50K. What is the original temperature of sink:-
&nbsp;<\/div>\n","suffix":null,"testcases":[],"solution":null,"insert_time":"2021-02-05
12:03:10","login_id":"admin","topic_id":0,"hint":null,"qmock_list":null,"unattempt":"2","unattempt_avg_time":"1
7.67","unevaluate":"0","unevaluate_avg_time":"45","correct":"0","correct_avg_time":"45","incorrect":"0","incorr
ect_avg_time":"45","sp_answer":"","mco_marks":"0","exam_ques":"no","exam_info":null,"sub_type":null,"is_bon
us":"0","is_extra_question":"no","answer_type":null,"integer_length":null,"error_status":"0","source_q_id":"1466
720","topic":"miscellaneous","subject":"physics","options":["325 K&nbsp;","375 K&nbsp;","300 K&nbsp;","350
K&nbsp;"]}]}}},{"id":"911232","section_id":"45863","sequence_id":"17","sequence_oid":null,"render_gid":"","num
_ques":"1","header":null,"content":null,"exam_id":"80","topic_id":"0","is_linked":null,"autoHeader":"","qdata":{"
17":{"subquestion":[{"id":"993308","group_id":"911232","sequence_id":"17","sequence_oid":null,"sequence_sid":
"0"

,"render_qid":"17","render_sqid":"","header":null,"type":"objective","category":"none","is_mco":"no","level":"lo
w","time":"120","marks":"4","neg_marks":"1","meta_id":"3353","dirty":null,"comment":null,"history":"[<b>Feb
05 2021 12:03:10][admin<\/b>][I][<b>admin<\/b>]","answer":"3","prefix":null,"content":"<div style=\"text-
align:justify\">Two carnot engines A and B are operated in succession. The first one, A receives heat from a source
at T<sub>1<\/sub> = 800K and rejects to sink at T<sub>2<\/sub>K. The second engine B receives heat rejected by
the first engine and rejects to another sink at T<sub>3<\/sub> = 300K. If the work outputs of two engines are
equal, then the value of T<sub>2<\/sub> is :-
<\/div>\n","suffix":null,"testcases":[],"solution":null,"insert_time":"2021-02-05
12:03:10","login_id":"admin","topic_id":0,"hint":null,"qmock_list":null,"unattempt":"2","unattempt_avg_time":"1
7.67","unevaluate":"0","unevaluate_avg_time":"45","correct":"0","correct_avg_time":"45","incorrect":"0","incorr
ect_avg_time":"45","sp_answer":"","mco_marks":"0","exam_ques":"no","exam_info":null,"sub_type":null,"is_bon
us":"0","is_extra_question":"no","answer_type":null,"integer_length":null,"error_status":"0","source_q_id":"1466
726","topic":"miscellaneous","subject":"physics","options":["100 K","300 K","550 K","700
K"]}]}}},{"id":"911233","section_id":"45863","sequence_id":"18","sequence_oid":null,"render_gid":"","num_ques"
:"1","header":null,"content":null,"exam_id":"80","topic_id":"0","is_linked":null,"autoHeader":"","qdata":{"18":{"s
ubquestion":[{"id":"993309","group_id":"911233","sequence_id":"18","sequence_oid":null,"sequence_sid":"0"

,"render_qid":"18","render_sqid":"","header":null,"type":"objective","category":"none","is_mco":"no","level":"me
dium","time":"120","marks":"4","neg_marks":"1","meta_id":"3353","dirty":null,"comment":null,"history":"[<b>Fe
b 05 2021 12:03:11][admin<\/b>][I][<b>admin<\/b>]","answer":"2","prefix":null,"content":"A cyclic process ABCA
is shown in the P&ndash;T diagram. Which of the following curves correctly represents the process on a V-T
diagram :-\n<div style=\"text-align:center\"><img
src=\"https:\/\/d2lbh14zkcqlst.cloudfront.net\/content_data\/content_images\/1612264425718319361phpbDyCX
Vimage.png\" style=\"vertical-align: middle;height:118px; width:140px\"
\/><\/div>\n","suffix":null,"testcases":[],"solution":null,"insert_time":"2021-02-05
12:03:11","login_id":"admin","topic_id":0,"hint":null,"qmock_list":null,"unattempt":"2","unattempt_avg_time":"1
9.33","unevaluate":"0","unevaluate_avg_time":"45","correct":"0","correct_avg_time":"45","incorrect":"0","incorr
ect_avg_time":"45","sp_answer":"","mco_marks":"0","exam_ques":"no","exam_info":null,"sub_type":null,"is_bon
us":"0","is_extra_question":"no","answer_type":null,"integer_length":null,"error_status":"0","source_q_id":"1466
744","topic":"miscellaneous","subject":"physics","options":["<img
src=\"https:\/\/d2lbh14zkcqlst.cloudfront.net\/content_data\/content_images\/16122644781699879898phpXmq
zQ6image.png\" style=\"vertical-align: middle;height:128px; width:150px\" \/>","<img
src=\"https:\/\/d2lbh14zkcqlst.cloudfront.net\/content_data\/content_images\/1612264501609518490phpRIjOZv
image.png\" style=\"vertical-align: middle;height:132px; width:150px\" \/>","<img
src=\"https:\/\/d2lbh14zkcqlst.cloudfront.net\/content_data\/content_images\/16122645381860569697php9teb
kximage.png\" style=\"vertical-align: middle;height:139px; width:150px\" \/>","<img
src=\"https:\/\/d2lbh14zkcqlst.cloudfront.net\/content_data\/content_images\/1612264555876296439phpReqsE
dimage.png\" style=\"vertical-align: middle;height:135px; width:150px\"
\/>"]}]}}},{"id":"911234","section_id":"45863","sequence_id":"19","sequence_oid":null,"render_gid":"","num_que
s":"1","header":null,"content":null,"exam_id":"80","topic_id":"0","is_linked":null,"autoHeader":"","qdata":{"19":{"
subquestion":[{"id":"993310","group_id":"911234","sequence_id":"19","sequence_oid":null,"sequence_sid":"0"
,"render_qid":"19","render_sqid":"","header":null,"type":"objective","category":"none","is_mco":"no","level":"lo
w","time":"120","marks":"4","neg_marks":"1","meta_id":"3353","dirty":null,"comment":null,"history":"[<b>Feb
05 2021 12:03:13][admin<\/b>][I][<b>admin<\/b>]","answer":"2","prefix":null,"content":"At what depth from
surface of earth weight of a particle becomes 20% of it&#39;s weight at surface of
earth","suffix":null,"testcases":[],"solution":null,"insert_time":"2021-02-05
12:03:13","login_id":"admin","topic_id":0,"hint":null,"qmock_list":null,"unattempt":"2","unattempt_avg_time":"1
9","unevaluate":"0","unevaluate_avg_time":"45","correct":"0","correct_avg_time":"45","incorrect":"0","incorrect
_avg_time":"45","sp_answer":"","mco_marks":"0","exam_ques":"no","exam_info":null,"sub_type":null,"is_bonus"
:"0","is_extra_question":"no","answer_type":null,"integer_length":null,"error_status":"0","source_q_id":"146675
1","topic":"miscellaneous","subject":"physics","options":["<img alt=\"frac{R}{5}\"
src=\"https:\/\/d2lbh14zkcqlst.cloudfront.net\/content_data\/content_images\/1612264660629264831.png\"
\/>","<img alt=\"frac{{4R}}{5}\"
src=\"https:\/\/d2lbh14zkcqlst.cloudfront.net\/content_data\/content_images\/1612264667139554287.png\"
\/>","<img alt=\"frac{{2R}}{5}\"
src=\"https:\/\/d2lbh14zkcqlst.cloudfront.net\/content_data\/content_images\/16122646731545715633.png\"
\/>","<img alt=\"frac{{3R}}{5}\"
src=\"https:\/\/d2lbh14zkcqlst.cloudfront.net\/content_data\/content_images\/16122646791539097762.png\"
\/>"]}]}}},{"id":"911235","section_id":"45863","sequence_id":"20","sequence_oid":null,"render_gid":"","num_que
s":"1","header":null,"content":null,"exam_id":"80","topic_id":"0","is_linked":null,"autoHeader":"","qdata":{"20":{"
subquestion":[{"id":"993311","group_id":"911235","sequence_id":"20","sequence_oid":null,"sequence_sid":"0

","render_qid":"20","render_sqid":"","header":null,"type":"objective","category":"none","is_mco":"no","level":"lo
w","time":"120","marks":"4","neg_marks":"1","meta_id":"3353","dirty":null,"comment":null,"history":"[<b>Feb
05 2021 12:03:15][admin<\/b>][I][<b>admin<\/b>]","answer":"1","prefix":null,"content":"<div style=\"text-
align:justify\">Acceleration due to gravity on moon is&nbsp;<img alt=\"frac{1}{6}\"
src=\"https:\/\/d2lbh14zkcqlst.cloudfront.net\/content_data\/content_images\/16122647241666276374.png\"
\/>&nbsp;of the acceleration due to gravity on earth. If the ratio of densities of earth (&rho;<sub>e<\/sub>) and
moon (&rho;<sub>m<\/sub>) is&nbsp;<img alt=\"left( {frac{{{rho _e}}}{{{rho _m}}}} right) = frac{5}{3}\"
src=\"https:\/\/d2lbh14zkcqlst.cloudfront.net\/content_data\/content_images\/1612264759133273919.png\"
\/>&nbsp;&nbsp;then ratio of radius of earth and moon is :-
&nbsp;<\/div>\n","suffix":null,"testcases":[],"solution":null,"insert_time":"2021-02-05
12:03:15","login_id":"admin","topic_id":0,"hint":null,"qmock_list":null,"unattempt":"2","unattempt_avg_time":"1
8","unevaluate":"0","unevaluate_avg_time":"45","correct":"0","correct_avg_time":"45","incorrect":"0","incorrect
_avg_time":"45","sp_answer":"","mco_marks":"0","exam_ques":"no","exam_info":null,"sub_type":null,"is_bonus"
:"0","is_extra_question":"no","answer_type":null,"integer_length":null,"error_status":"0","source_q_id":"146676
0","topic":"miscellaneous","subject":"physics","options":["<img alt=\"frac{{18}}{5}\"
src=\"https:\/\/d2lbh14zkcqlst.cloudfront.net\/content_data\/content_images\/1612264769781022671.png\"
\/>","<img alt=\"frac{6}{1}\"
src=\"https:\/\/d2lbh14zkcqlst.cloudfront.net\/content_data\/content_images\/1612264775538436961.png\"
\/>","<img alt=\"frac{5}{2}\"
src=\"https:\/\/d2lbh14zkcqlst.cloudfront.net\/content_data\/content_images\/1612264780296730757.png\"
\/>","<img alt=\"frac{{2sqrt 3 }}{1}\"
src=\"https:\/\/d2lbh14zkcqlst.cloudfront.net\/content_data\/content_images\/1612264785370438834.png\"
\/>"]}]}}}]},{"id":"45864","paper_id":"12803","section":"Physics","subsection":"SEC-
B","sequence_id":"21","sequence_sid":"0","header":"<div>\r\n<ul>\r\n\t<li>This section has <b>TEN<\/b>
questions.<\/li>\r\n\t<li>Candidates have to <b>attempt any 5 questions out of 10<\/b>. If more than 5 questions
are attempted, then only first 5 attempted questions will be evaluated.<\/li>\r\n\t<li>The answer to each question
is a <b>NUMERICAL VALUE<\/b>.<\/li>\r\n\t<li>For each question, enter the correct numerical value (in decimal
notation, truncated\/rounded-off to the <b>second decimal place <\/b>e.g. 6.25, 7.00, &ndash;0.33, &ndash;.30,
30.27, &ndash;127.30, if answer is 11.36777..... then both 11.36 and 11.37 will be correct).<\/li>\r\n\t<li>For each
question, type the correct integer in the space provided below the question using provided number
keys.<\/li>\r\n\t<li><b>Marking scheme:<\/b>\r\n\t<ul>\r\n\t\t<li>Full Marks : +4 If the correct integer is typed in
the provided space.<\/li>\r\n\t\t<li>Zero Marks : 0 In all other
cases.<\/li>\r\n\t<\/ul>\r\n\t<\/li>\r\n<\/ul>\r\n<\/div>\r\n","num_grps":"10","marks":"20","neg_marks":"0","ti
me_duration":"20","type":"paper","ans_limit":"5","gdata":[{"id":"911236","section_id":"45864","sequence_id":"2
1","sequence_oid":null,"render_gid":"","num_ques":"1","header":null,"content":null,"exam_id":"80","topic_id":"0
","is_linked":null,"autoHeader":"","qdata":{"21":{"subquestion":[{"id":"993312","group_id":"911236","sequence_i
d":"21","sequence_oid":null,"sequence_sid":"0"

,"render_qid":"21","render_sqid":"","header":null,"type":"subjective","category":"none","is_mco":"no","level":"m
edium","time":"120","marks":"4","neg_marks":"0","meta_id":"3353","dirty":null,"comment":null,"history":"[<b>F
eb 05 2021 12:03:15][admin<\/b>][I][<b>admin<\/b>]","answer":"4.00","prefix":null,"content":"The wavelength of
a wave is 99 cm and that of other is 100 cm. Speed of sound is&nbsp;396 m\/s. The number of beats heard is :-
","suffix":null,"testcases":[],"solution":null,"insert_time":"2021-02-05
12:03:15","login_id":"admin","topic_id":0,"hint":null,"qmock_list":null,"unattempt":"0","unattempt_avg_time":"4
5","unevaluate":"0","unevaluate_avg_time":"45","correct":"2","correct_avg_time":"23.33","incorrect":"0","incorr
ect_avg_time":"45","sp_answer":"","mco_marks":"0","exam_ques":"no","exam_info":null,"sub_type":"numerical"
,"is_bonus":"0","is_extra_question":"no","answer_type":null,"integer_length":null,"error_status":"0","source_q_i
d":"1466809","topic":"miscellaneous","subject":"physics"}]}}},{"id":"911237","section_id":"45864","sequence_id":
"22","sequence_oid":null,"render_gid":"","num_ques":"1","header":null,"content":null,"exam_id":"80","topic_id":
"0","is_linked":null,"autoHeader":"","qdata":{"22":{"subquestion":[{"id":"993313","group_id":"911237","sequence
_id":"22","sequence_oid":null,"sequence_sid":"0

","render_qid":"22","render_sqid":"","header":null,"type":"subjective","category":"none","is_mco":"no","level":"h
igh","time":"120","marks":"4","neg_marks":"0","meta_id":"3353","dirty":null,"comment":null,"history":"[<b>Feb
05 2021 12:03:15][admin<\/b>][I][<b>admin<\/b>]","answer":"5.00","prefix":null,"content":"<div style=\"text-
align:justify\">A bus is moving with a velocity of 5 m\/s towards a huge wall. The driver sounds a horn of frequency
165Hz. If the speed of sound in air is 335 m\/s, then &nbsp;the number &nbsp;of beats per second heard by a
passenger on bus will be &ndash;&nbsp;<\/div>\n","suffix":null,"testcases":[],"solution":null,"insert_time":"2021-
02-05
12:03:15","login_id":"admin","topic_id":0,"hint":null,"qmock_list":null,"unattempt":"2","unattempt_avg_time":"1
6","unevaluate":"0","unevaluate_avg_time":"45","correct":"0","correct_avg_time":"45","incorrect":"0","incorrect
_avg_time":"45","sp_answer":"","mco_marks":"0","exam_ques":"no","exam_info":null,"sub_type":"numerical","is
_bonus":"0","is_extra_question":"no","answer_type":null,"integer_length":null,"error_status":"0","source_q_id":"
1466816","topic":"miscellaneous","subject":"physics"}]}}},{"id":"911238","section_id":"45864","sequence_id":"23"
,"sequence_oid":null,"render_gid":"","num_ques":"1","header":null,"content":null,"exam_id":"80","topic_id":"0",
"is_linked":null,"autoHeader":"","qdata":{"23":{"subquestion":[{"id":"993314","group_id":"911238","sequence_id
":"23","sequence_oid":null,"sequence_sid":"0

","render_qid":"23","render_sqid":"","header":null,"type":"subjective","category":"none","is_mco":"no","level":"
medium","time":"120","marks":"4","neg_marks":"0","meta_id":"3353","dirty":null,"comment":null,"history":"[<b>
Feb 05 2021 12:03:15][admin<\/b>][I][<b>admin<\/b>]","answer":"5.00","prefix":null,"content":"The root mean
square speed of 4 molecules of a gas having speeds 1, 2, 3, 4 ms<sup>&ndash;1<\/sup>, is&nbsp;<img alt=\"Xsqrt
{0.3} \"
src=\"https:\/\/d2lbh14zkcqlst.cloudfront.net\/content_data\/content_images\/16122652661537720787.png\"
\/>.&nbsp;. Find X.","suffix":null,"testcases":[],"solution":null,"insert_time":"2021-02-05
12:03:15","login_id":"admin","topic_id":0,"hint":null,"qmock_list":null,"unattempt":"2","unattempt_avg_time":"1
6.67","unevaluate":"0","unevaluate_avg_time":"45","correct":"0","correct_avg_time":"45","incorrect":"0","incorr
ect_avg_time":"45","sp_answer":"","mco_marks":"0","exam_ques":"no","exam_info":null,"sub_type":"numerical"
,"is_bonus":"0","is_extra_question":"no","answer_type":null,"integer_length":null,"error_status":"0","source_q_i
d":"1466823","topic":"miscellaneous","subject":"physics"}]}}},{"id":"911239","section_id":"45864","sequence_id":
"24","sequence_oid":null,"render_gid":"","num_ques":"1","header":null,"content":null,"exam_id":"80","topic_id":
"0","is_linked":null,"autoHeader":"","qdata":{"24":{"subquestion":[{"id":"993315","group_id":"911239","sequence
_id":"24","sequence_oid":null,"sequence_sid":"0"

,"render_qid":"24","render_sqid":"","header":null,"type":"subjective","category":"none","is_mco":"no","level":"m
edium","time":"120","marks":"4","neg_marks":"0","meta_id":"3353","dirty":null,"comment":null,"history":"[<b>F
eb 05 2021 12:03:15][admin<\/b>][I][<b>admin<\/b>]","answer":"5.00","prefix":null,"content":"If one mole
monoatomic (f = 3), 2 mol diatomic (f = 5) and 2 mole triatomic (f = 6) are mixed, find the f<sub>equivalent<\/sub>
of mixture(f = Degrees of freedom of gas) :-","suffix":null,"testcases":[],"solution":null,"insert_time":"2021-02-05
12:03:15","login_id":"admin","topic_id":0,"hint":null,"qmock_list":null,"unattempt":"2","unattempt_avg_time":"1
7","unevaluate":"0","unevaluate_avg_time":"45","correct":"0","correct_avg_time":"45","incorrect":"0","incorrect
_avg_time":"45","sp_answer":"","mco_marks":"0","exam_ques":"no","exam_info":null,"sub_type":"numerical","is
_bonus":"0","is_extra_question":"no","answer_type":null,"integer_length":null,"error_status":"0","source_q_id":"
1466829","topic":"miscellaneous","subject":"physics"}]}}},{"id":"911240","section_id":"45864","sequence_id":"25"
,"sequence_oid":null,"render_gid":"","num_ques":"1","header":null,"content":null,"exam_id":"80","topic_id":"0",
"is_linked":null,"autoHeader":"","qdata":{"25":{"subquestion":[{"id":"993316","group_id":"911240","sequence_id
":"25","sequence_oid":null,"sequence_sid":"0"

,"render_qid":"25","render_sqid":"","header":null,"type":"subjective","category":"none","is_mco":"no","level":"m
edium","time":"120","marks":"4","neg_marks":"0","meta_id":"3353","dirty":null,"comment":null,"history":"[<b>F
eb 05 2021 12:03:16][admin<\/b>][I][<b>admin<\/b>]","answer":"6.00","prefix":null,"content":"<div style=\"text-
align:justify\">An ideal gas (&gamma; = 3\/2)at a pressures of 1 atmosphere and temperature of 27&ordm;C is
compressed adiabatically until its pressure becomes 8 times the initial pressure, then the final temperature is
100X(In Kelvin) find X.&nbsp;<\/div>\n","suffix":null,"testcases":[],"solution":null,"insert_time":"2021-02-05
12:03:16","login_id":"admin","topic_id":0,"hint":null,"qmock_list":null,"unattempt":"2","unattempt_avg_time":"1
7","unevaluate":"0","unevaluate_avg_time":"45","correct":"0","correct_avg_time":"45","incorrect":"0","incorrect
_avg_time":"45","sp_answer":"","mco_marks":"0","exam_ques":"no","exam_info":null,"sub_type":"numerical","is
_bonus":"0","is_extra_question":"no","answer_type":null,"integer_length":null,"error_status":"0","source_q_id":"
1466837","topic":"miscellaneous","subject":"physics"}]}}},{"id":"911241","section_id":"45864","sequence_id":"26"
,"sequence_oid":null,"render_gid":"","num_ques":"1","header":null,"content":null,"exam_id":"80","topic_id":"0",
"is_linked":null,"autoHeader":"","qdata":{"26":{"subquestion":[{"id":"993317","group_id":"911241","sequence_id
":"26","sequence_oid":null,"sequence_sid":"0"

,"render_qid":"26","render_sqid":"","header":null,"type":"subjective","category":"none","is_mco":"no","level":"m
edium","time":"120","marks":"4","neg_marks":"0","meta_id":"3353","dirty":null,"comment":null,"history":"[<b>F
eb 05 2021 12:03:16][admin<\/b>][I][<b>admin<\/b>]","answer":"4.00","prefix":null,"content":"The pressure-
volume graph of a gas is as shown find the total work done (in J) in 1 cycle.\n<div style=\"text-align:center\"><img
src=\"https:\/\/d2lbh14zkcqlst.cloudfront.net\/content_data\/content_images\/1612265432342489320phpLlScO
Kimage.png\" style=\"vertical-align: middle;height:152px; width:180px\"
\/><\/div>\n","suffix":null,"testcases":[],"solution":null,"insert_time":"2021-02-05
12:03:16","login_id":"admin","topic_id":0,"hint":null,"qmock_list":null,"unattempt":"2","unattempt_avg_time":"1
6.33","unevaluate":"0","unevaluate_avg_time":"45","correct":"0","correct_avg_time":"45","incorrect":"0","incorr
ect_avg_time":"45","sp_answer":"","mco_marks":"0","exam_ques":"no","exam_info":null,"sub_type":"numerical"
,"is_bonus":"0","is_extra_question":"no","answer_type":null,"integer_length":null,"error_status":"0","source_q_i
d":"1466848","topic":"miscellaneous","subject":"physics"}]}}},{"id":"911242","section_id":"45864","sequence_id":
"27","sequence_oid":null,"render_gid":"","num_ques":"1","header":null,"content":null,"exam_id":"80","topic_id":
"0","is_linked":null,"autoHeader":"","qdata":{"27":{"subquestion":[{"id":"993318","group_id":"911242","sequence
_id":"27","sequence_oid":null,"sequence_sid":"0

","render_qid":"27","render_sqid":"","header":null,"type":"subjective","category":"none","is_mco":"no","level":"l
ow","time":"120","marks":"4","neg_marks":"0","meta_id":"3353","dirty":null,"comment":null,"history":"[<b>Feb
05 2021 12:03:16][admin<\/b>][I][<b>admin<\/b>]","answer":"8.00","prefix":null,"content":"<div style=\"text-
align:justify\">A Carnot engine operates with source at 127&ordm;C and sink at 27&ordm;C. If the source supplies
32 kJ of heat energy, the work done by the engine (kJ)is-
&nbsp;<\/div>\n","suffix":null,"testcases":[],"solution":null,"insert_time":"2021-02-05
12:03:16","login_id":"admin","topic_id":0,"hint":null,"qmock_list":null,"unattempt":"2","unattempt_avg_time":"1
6.67","unevaluate":"0","unevaluate_avg_time":"45","correct":"0","correct_avg_time":"45","incorrect":"0","incorr
ect_avg_time":"45","sp_answer":"","mco_marks":"0","exam_ques":"no","exam_info":null,"sub_type":"numerical"
,"is_bonus":"0","is_extra_question":"no","answer_type":null,"integer_length":null,"error_status":"0","source_q_i
d":"1466853","topic":"miscellaneous","subject":"physics"}]}}},{"id":"911243","section_id":"45864","sequence_id":
"28","sequence_oid":null,"render_gid":"","num_ques":"1","header":null,"content":null,"exam_id":"80","topic_id":
"0","is_linked":null,"autoHeader":"","qdata":{"28":{"subquestion":[{"id":"993319","group_id":"911243","sequence
_id":"28","sequence_oid":null,"sequence_sid":"0"

,"render_qid":"28","render_sqid":"","header":null,"type":"subjective","category":"none","is_mco":"no","level":"lo
w","time":"120","marks":"4","neg_marks":"0","meta_id":"3353","dirty":null,"comment":null,"history":"[<b>Feb
05 2021 12:03:16][admin<\/b>][I][<b>admin<\/b>]","answer":"0.00","prefix":null,"content":"A mass of 5 kg is
placed inside a spherical shell of mass 40 kg and radius 1m.&nbsp;Given : G &nbsp;= 1 unit (SI). The force on 5 kg
mass is :","suffix":null,"testcases":[],"solution":null,"insert_time":"2021-02-05
12:03:16","login_id":"admin","topic_id":0,"hint":null,"qmock_list":null,"unattempt":"2","unattempt_avg_time":"1
6.33","unevaluate":"0","unevaluate_avg_time":"45","correct":"0","correct_avg_time":"45","incorrect":"0","incorr
ect_avg_time":"45","sp_answer":"","mco_marks":"0","exam_ques":"no","exam_info":null,"sub_type":"numerical"
,"is_bonus":"0","is_extra_question":"no","answer_type":null,"integer_length":null,"error_status":"0","source_q_i
d":"1466855","topic":"miscellaneous","subject":"physics"}]}}},{"id":"911244","section_id":"45864","sequence_id":
"29","sequence_oid":null,"render_gid":"","num_ques":"1","header":null,"content":null,"exam_id":"80","topic_id":
"0","is_linked":null,"autoHeader":"","qdata":{"29":{"subquestion":[{"id":"993320","group_id":"911244","sequence
_id":"29","sequence_oid":null,"sequence_sid":"0

","render_qid":"29","render_sqid":"","header":null,"type":"subjective","category":"none","is_mco":"no","level":"l
ow","time":"120","marks":"4","neg_marks":"0","meta_id":"3353","dirty":null,"comment":null,"history":"[<b>Feb
05 2021 12:03:16][admin<\/b>][I][<b>admin<\/b>]","answer":"5.00","prefix":null,"content":"A particle of mass 8
kg is placed at a height 3R from surface of earth, where R is radius of earth. Find weight (N) of particle at that
point, if g on the surface of earth = 10m\/s<sup>2<\/sup>
?","suffix":null,"testcases":[],"solution":null,"insert_time":"2021-02-05
12:03:16","login_id":"admin","topic_id":0,"hint":null,"qmock_list":null,"unattempt":"2","unattempt_avg_time":"1
7.33","unevaluate":"0","unevaluate_avg_time":"45","correct":"0","correct_avg_time":"45","incorrect":"0","incorr
ect_avg_time":"45","sp_answer":"","mco_marks":"0","exam_ques":"no","exam_info":null,"sub_type":"numerical"
,"is_bonus":"0","is_extra_question":"no","answer_type":null,"integer_length":null,"error_status":"0","source_q_i
d":"1466861","topic":"miscellaneous","subject":"physics"}]}}},{"id":"911245","section_id":"45864","sequence_id":
"30","sequence_oid":null,"render_gid":"","num_ques":"1","header":null,"content":null,"exam_id":"80","topic_id":
"0","is_linked":null,"autoHeader":"","qdata":{"30":{"subquestion":[{"id":"993321","group_id":"911245","sequence
_id":"30","sequence_oid":null,"sequence_sid":"0

","render_qid":"30","render_sqid":"","header":null,"type":"subjective","category":"none","is_mco":"no","level":"
medium","time":"120","marks":"4","neg_marks":"0","meta_id":"3353","dirty":null,"comment":null,"history":"[<b>
Feb 05 2021 12:03:16][admin<\/b>][I][<b>admin<\/b>]","answer":"4.00","prefix":null,"content":"<div style=\"text-
align:justify\">Velocity of a planet revolving around sun at perigee (closest point) is 400 m\/sec and at
apogee&nbsp;(farthest point) is 100 m\/sec. Then find ratio of maximum distance to minimum distance of planet
from&nbsp;sun-<\/div>\n","suffix":null,"testcases":[],"solution":null,"insert_time":"2021-02-05
12:03:16","login_id":"admin","topic_id":0,"hint":null,"qmock_list":null,"unattempt":"2","unattempt_avg_time":"1
6.33","unevaluate":"0","unevaluate_avg_time":"45","correct":"0","correct_avg_time":"45","incorrect":"0","incorr
ect_avg_time":"45","sp_answer":"","mco_marks":"0","exam_ques":"no","exam_info":null,"sub_type":"numerical"
,"is_bonus":"0","is_extra_question":"no","answer_type":null,"integer_length":null,"error_status":"0","source_q_i
d":"1466866","topic":"miscellaneous","subject":"physics"}]}}}]}]},"Chemistry":{"subsections":[{"id":"45865","paper
_id":"12803","section":"Chemistry","subsection":"SEC-
A","sequence_id":"31","sequence_sid":"0","header":"<ul>\r\n\t<li>This section contains <b>TWENTY<\/b>
questions.<\/li>\r\n\t<li>Each question has FOUR options (A), (B), (C) and (D). <b>ONLY ONE<\/b> of these four
options is correct.<\/li>\r\n\t<li>For each question, select the alphabet corresponding to the correct option
provided below the questions.<\/li>\r\n\t<li><b>Marking scheme:<\/b>&nbsp;\r\n\t<ul>\r\n\t\t<li>Full Marks :
<span style=\"color:black\">+4 If correct answer is selected.<\/span><\/li>\r\n\t\t<li><span
style=\"color:black\">Zero Marks <\/span>: <span style=\"color:black\">0&nbsp;If none of the option is
selected.<\/span><\/li>\r\n\t\t<li><span style=\"color:black\">Negative Marks <\/span>: <span
style=\"color:black\">&ndash;1 If wrong option is
selected.<\/span><\/li>\r\n\t<\/ul>\r\n\t<\/li>\r\n<\/ul>\r\n","num_grps":"20","marks":"80","neg_marks":"0","ti
me_duration":"40","type":"paper","ans_limit":"0","gdata":[{"id":"911246","section_id":"45865","sequence_id":"3
1","sequence_oid":null,"render_gid":"","num_ques":"1","header":null,"content":null,"exam_id":"80","topic_id":"0
","is_linked":null,"autoHeader":"","qdata":{"31":{"subquestion":[{"id":"993322","group_id":"911246","sequence_i
d":"31","sequence_oid":null,"sequence_sid":"0"

,"render_qid":"31","render_sqid":"","header":null,"type":"objective","category":"none","is_mco":"no","level":"me
dium","time":"120","marks":"4","neg_marks":"1","meta_id":"3390","dirty":null,"comment":null,"history":"[<b>Fe
b 05 2021 12:03:16][admin<\/b>][I][<b>admin<\/b>]","answer":"1","prefix":null,"content":"In the given heating
sequence-<br \/>\n<img
src=\"https:\/\/d2lbh14zkcqlst.cloudfront.net\/content_data\/content_images\/16122660821538314027phpmttn
sEimage.png\" style=\"vertical-align: middle;height:36px; width:350px\" \/><br \/>\nthen x and y respectively are-
","suffix":null,"testcases":[],"solution":null,"insert_time":"2021-02-05
12:03:16","login_id":"admin","topic_id":0,"hint":null,"qmock_list":null,"unattempt":"1","unattempt_avg_time":"2
7","unevaluate":"0","unevaluate_avg_time":"45","correct":"1","correct_avg_time":"38.5","incorrect":"0","incorre
ct_avg_time":"45","sp_answer":"","mco_marks":"0","exam_ques":"no","exam_info":null,"sub_type":null,"is_bonu
s":"0","is_extra_question":"no","answer_type":null,"integer_length":null,"error_status":"0","source_q_id":"14669
05","topic":"miscellaneous","subject":"chemistry","options":["x = Metaboric acid and y = Tetraboric acid.","x =
Tetraboric acid and y = Metaboric acid.","x = Borax and y = Metaboric acid.","x = Tetraboric acid and y =
Borax."]}]}}},{"id":"911247","section_id":"45865","sequence_id":"32","sequence_oid":null,"render_gid":"","num_q
ues":"1","header":null,"content":null,"exam_id":"80","topic_id":"0","is_linked":null,"autoHeader":"","qdata":{"32"
:{"subquestion":[{"id":"993323","group_id":"911247","sequence_id":"32","sequence_oid":null,"sequence_sid":"0"
,

"render_qid":"32","render_sqid":"","header":null,"type":"objective","category":"none","is_mco":"no","level":"low
","time":"120","marks":"4","neg_marks":"1","meta_id":"3390","dirty":null,"comment":null,"history":"[<b>Feb 05
2021 12:03:16][admin<\/b>][I][<b>admin<\/b>]","answer":"2","prefix":null,"content":"Identify the correct set of
statements regarding ortho-boric acid-<br \/>\n(i) It is a 3D extended network solid.&nbsp;&nbsp; &nbsp;<br
\/>\n(ii) &nbsp;It is a weak protic mono-basic acid.<br \/>\n(iii) It is a weak aprotic mono-basic acid.&nbsp;&nbsp;
&nbsp;<br \/>\n(iv) It contains depole-dipole interactions.<br \/>\n(v) It has B and O atoms with sp2 hybridisation
each.<br \/>\n(vi) It is a 2D sheet network solid.","suffix":null,"testcases":[],"solution":null,"insert_time":"2021-02-
05
12:03:16","login_id":"admin","topic_id":0,"hint":null,"qmock_list":null,"unattempt":"2","unattempt_avg_time":"1
8.67","unevaluate":"0","unevaluate_avg_time":"45","correct":"0","correct_avg_time":"45","incorrect":"0","incorr
ect_avg_time":"45","sp_answer":"","mco_marks":"0","exam_ques":"no","exam_info":null,"sub_type":null,"is_bon
us":"0","is_extra_question":"no","answer_type":null,"integer_length":null,"error_status":"0","source_q_id":"1466
929","topic":"miscellaneous","subject":"chemistry","options":["(ii), (iv), (v) (vi)","(iii), (iv), (v) (vi)","(i), (ii), (v)
(vi)","(ii), (iii), (iv)
(vi)"]}]}}},{"id":"911248","section_id":"45865","sequence_id":"33","sequence_oid":null,"render_gid":"","num_que
s":"1","header":null,"content":null,"exam_id":"80","topic_id":"0","is_linked":null,"autoHeader":"","qdata":{"33":{"
subquestion":[{"id":"993324","group_id":"911248","sequence_id":"33","sequence_oid":null,"sequence_sid":"0
","render_qid":"33","render_sqid":"","header":null,"type":"objective","category":"none","is_mco":"no","level":"lo
w","time":"120","marks":"4","neg_marks":"1","meta_id":"3390","dirty":null,"comment":null,"history":"[<b>Feb
05 2021 12:03:16][admin<\/b>][I][<b>admin<\/b>]","answer":"3","prefix":null,"content":"Borax is actually made
of two tetrahedral and two triangular units joined together and should be written as-<br
\/>\nNa<sub>2<\/sub>[B<sub>4<\/sub>O<sub>5<\/sub>(OH)<sub>4<\/sub>].8H<sub>2<\/sub>O<br
\/>\nConsider the following statements about borax-<br \/>\n(i) Each boron atom has four B &ndash; O bonds.<br
\/>\n(ii) Each boron atom has only three B &ndash; O bonds.<br \/>\n(iii) Two boron atoms have four B &ndash; O
bonds while two have three B &ndash; O bonds.<br \/>\n(iv) Each boron atom has one &ndash;OH group.<br
\/>\nSelect correct set of statements-","suffix":null,"testcases":[],"solution":null,"insert_time":"2021-02-05
12:03:16","login_id":"admin","topic_id":0,"hint":null,"qmock_list":null,"unattempt":"2","unattempt_avg_time":"1
8.33","unevaluate":"0","unevaluate_avg_time":"45","correct":"0","correct_avg_time":"45","incorrect":"0","incorr
ect_avg_time":"45","sp_answer":"","mco_marks":"0","exam_ques":"no","exam_info":null,"sub_type":null,"is_bon
us":"0","is_extra_question":"no","answer_type":null,"integer_length":null,"error_status":"0","source_q_id":"1466
943","topic":"miscellaneous","subject":"chemistry","options":["(i), (ii)","(ii), (iii)","(iii), (iv)","(i),
(iii)"]}]}}},{"id":"911249","section_id":"45865","sequence_id":"34","sequence_oid":null,"render_gid":"","num_que
s":"1","header":null,"content":null,"exam_id":"80","topic_id":"0","is_linked":null,"autoHeader":"","qdata":{"34":{"
subquestion":[{"id":"993325","group_id":"911249","sequence_id":"34","sequence_oid":null,"sequence_sid":"0

","render_qid":"34","render_sqid":"","header":null,"type":"objective","category":"none","is_mco":"no","level":"hi
gh","time":"120","marks":"4","neg_marks":"1","meta_id":"3390","dirty":null,"comment":null,"history":"[<b>Feb
05 2021 12:03:16][admin<\/b>][I][<b>admin<\/b>]","answer":"2","prefix":null,"content":"The total number of
isomers possible for disubstituted borazene B<sub>3<\/sub>N<sub>3<\/sub>H<sub>4<\/sub>X<sub>2<\/sub> is-
&nbsp;","suffix":null,"testcases":[],"solution":null,"insert_time":"2021-02-05
12:03:17","login_id":"admin","topic_id":0,"hint":null,"qmock_list":null,"unattempt":"2","unattempt_avg_time":"1
6.33","unevaluate":"0","unevaluate_avg_time":"45","correct":"0","correct_avg_time":"45","incorrect":"0","incorr
ect_avg_time":"45","sp_answer":"","mco_marks":"0","exam_ques":"no","exam_info":null,"sub_type":null,"is_bon
us":"0","is_extra_question":"no","answer_type":null,"integer_length":null,"error_status":"0","source_q_id":"1466
947","topic":"miscellaneous","subject":"chemistry","options":["3","4","6","5"]}]}}},{"id":"911250","section_id":"45
865","sequence_id":"35","sequence_oid":null,"render_gid":"","num_ques":"1","header":null,"content":null,"exam
_id":"80","topic_id":"0","is_linked":null,"autoHeader":"","qdata":{"35":{"subquestion":[{"id":"993326","group_id":
"911250","sequence_id":"35","sequence_oid":null,"sequence_sid":"0

","render_qid":"35","render_sqid":"","header":null,"type":"objective","category":"none","is_mco":"no","level":"lo
w","time":"120","marks":"4","neg_marks":"1","meta_id":"3390","dirty":null,"comment":null,"history":"[<b>Feb
05 2021 12:03:17][admin<\/b>][I][<b>admin<\/b>]","answer":"2","prefix":null,"content":"Which of the following
pair of substances will not evolve H<sub>2<\/sub> gas
?","suffix":null,"testcases":[],"solution":null,"insert_time":"2021-02-05
12:03:17","login_id":"admin","topic_id":0,"hint":null,"qmock_list":null,"unattempt":"2","unattempt_avg_time":"1
6","unevaluate":"0","unevaluate_avg_time":"45","correct":"0","correct_avg_time":"45","incorrect":"0","incorrect
_avg_time":"45","sp_answer":"","mco_marks":"0","exam_ques":"no","exam_info":null,"sub_type":null,"is_bonus"
:"0","is_extra_question":"no","answer_type":null,"integer_length":null,"error_status":"0","source_q_id":"146695
1","topic":"miscellaneous","subject":"chemistry","options":["Iron and aqueous
H<sub>2<\/sub>SO<sub>4<\/sub>","Copper and HCl(aq)","Sodium and ethanol","Iron and
steam"]}]}}},{"id":"911251","section_id":"45865","sequence_id":"36","sequence_oid":null,"render_gid":"","num_q
ues":"1","header":null,"content":null,"exam_id":"80","topic_id":"0","is_linked":null,"autoHeader":"","qdata":{"36"
:{"subquestion":[{"id":"993327","group_id":"911251","sequence_id":"36","sequence_oid":null,"sequence_sid":"0"
,

"render_qid":"36","render_sqid":"","header":null,"type":"objective","category":"none","is_mco":"no","level":"me
dium","time":"120","marks":"4","neg_marks":"1","meta_id":"3390","dirty":null,"comment":null,"history":"[<b>Fe
b 05 2021 12:03:17][admin<\/b>][I][<b>admin<\/b>]","answer":"2","prefix":null,"content":"<div style=\"text-
align:justify\">An oranged coloured solution of K<sub>2<\/sub>Cr<sub>2<\/sub>O<sub>7<\/sub> acidified with
H<sub>2<\/sub>SO<sub>4<\/sub>, on treatment with H<sub>2<\/sub>O<sub>2<\/sub> gives a blue coloured
solution of (X). Find out the number of Cr &ndash; O single bonds in (x).
&nbsp;<\/div>\n","suffix":null,"testcases":[],"solution":null,"insert_time":"2021-02-05
12:03:17","login_id":"admin","topic_id":0,"hint":null,"qmock_list":null,"unattempt":"2","unattempt_avg_time":"1
6.67","unevaluate":"0","unevaluate_avg_time":"45","correct":"0","correct_avg_time":"45","incorrect":"0","incorr
ect_avg_time":"45","sp_answer":"","mco_marks":"0","exam_ques":"no","exam_info":null,"sub_type":null,"is_bon
us":"0","is_extra_question":"no","answer_type":null,"integer_length":null,"error_status":"0","source_q_id":"1466
963","topic":"miscellaneous","subject":"chemistry","options":["5","4","3","2"]}]}}},{"id":"911252","section_id":"45
865","sequence_id":"37","sequence_oid":null,"render_gid":"","num_ques":"1","header":null,"content":null,"exam
_id":"80","topic_id":"0","is_linked":null,"autoHeader":"","qdata":{"37":{"subquestion":[{"id":"993328","group_id":
"911252","sequence_id":"37","sequence_oid":null,"sequence_sid":"0

","render_qid":"37","render_sqid":"","header":null,"type":"objective","category":"none","is_mco":"no","level":"m
edium","time":"120","marks":"4","neg_marks":"1","meta_id":"3390","dirty":null,"comment":null,"history":"[<b>F
eb 05 2021 12:03:17][admin<\/b>][I][<b>admin<\/b>]","answer":"3","prefix":null,"content":"The green colour
produced in the borax bead test of a chromium (III) salt is due to-
&nbsp;","suffix":null,"testcases":[],"solution":null,"insert_time":"2021-02-05
12:03:17","login_id":"admin","topic_id":0,"hint":null,"qmock_list":null,"unattempt":"2","unattempt_avg_time":"1
6","unevaluate":"0","unevaluate_avg_time":"45","correct":"0","correct_avg_time":"45","incorrect":"0","incorrect
_avg_time":"45","sp_answer":"","mco_marks":"0","exam_ques":"no","exam_info":null,"sub_type":null,"is_bonus"
:"0","is_extra_question":"no","answer_type":null,"integer_length":null,"error_status":"0","source_q_id":"146697
0","topic":"miscellaneous","subject":"chemistry","options":["Cr
B","Cr<sub>2<\/sub>O<sub>3<\/sub>","Cr(BO<sub>2<\/sub>)<sub>3<\/sub>","Cr<sub>2<\/sub>(B<sub>4<\/sub>
O<sub>7<\/sub>)"]}]}}},{"id":"911253","section_id":"45865","sequence_id":"38","sequence_oid":null,"render_gid"
:"","num_ques":"1","header":null,"content":null,"exam_id":"80","topic_id":"0","is_linked":null,"autoHeader":"","q
data":{"38":{"subquestion":[{"id":"993329","group_id":"911253","sequence_id":"38","sequence_oid":null,"sequen
ce_sid":"0"

,"render_qid":"38","render_sqid":"","header":null,"type":"objective","category":"none","is_mco":"no","level":"lo
w","time":"120","marks":"4","neg_marks":"1","meta_id":"3390","dirty":null,"comment":null,"history":"[<b>Feb
05 2021 12:03:17][admin<\/b>][I][<b>admin<\/b>]","answer":"3","prefix":null,"content":"Hardness of water is due
to presence of &nbsp;which of the following species
?","suffix":null,"testcases":[],"solution":null,"insert_time":"2021-02-05
12:03:17","login_id":"admin","topic_id":0,"hint":null,"qmock_list":null,"unattempt":"2","unattempt_avg_time":"1
6","unevaluate":"0","unevaluate_avg_time":"45","correct":"0","correct_avg_time":"45","incorrect":"0","incorrect
_avg_time":"45","sp_answer":"","mco_marks":"0","exam_ques":"no","exam_info":null,"sub_type":null,"is_bonus"
:"0","is_extra_question":"no","answer_type":null,"integer_length":null,"error_status":"0","source_q_id":"146697
7","topic":"miscellaneous","subject":"chemistry","options":["CaCO<sub>3<\/sub>","BaSO<sub>4<\/sub>","Mg(HC
O<sub>3<\/sub>)<sub>2<\/sub>","CCl<sub>4<\/sub>"]}]}}},{"id":"911254","section_id":"45865","sequence_id":"3
9","sequence_oid":null,"render_gid":"","num_ques":"1","header":null,"content":null,"exam_id":"80","topic_id":"0
","is_linked":null,"autoHeader":"","qdata":{"39":{"subquestion":[{"id":"993330","group_id":"911254","sequence_i
d":"39","sequence_oid":null,"sequence_sid":"0

","render_qid":"39","render_sqid":"","header":null,"type":"objective","category":"none","is_mco":"no","level":"lo
w","time":"120","marks":"4","neg_marks":"1","meta_id":"3390","dirty":null,"comment":null,"history":"[<b>Feb
05 2021 12:03:17][admin<\/b>][I][<b>admin<\/b>]","answer":"3","prefix":null,"content":"Inorganic benzene is
:&nbsp;&nbsp; &nbsp;","suffix":null,"testcases":[],"solution":null,"insert_time":"2021-02-05
12:03:17","login_id":"admin","topic_id":0,"hint":null,"qmock_list":null,"unattempt":"2","unattempt_avg_time":"1
6","unevaluate":"0","unevaluate_avg_time":"45","correct":"0","correct_avg_time":"45","incorrect":"0","incorrect
_avg_time":"45","sp_answer":"","mco_marks":"0","exam_ques":"no","exam_info":null,"sub_type":null,"is_bonus"
:"0","is_extra_question":"no","answer_type":null,"integer_length":null,"error_status":"0","source_q_id":"146698
2","topic":"miscellaneous","subject":"chemistry","options":["non-planar","Homocyclic","Aromatic&nbsp;","Known
as
borax"]}]}}},{"id":"911255","section_id":"45865","sequence_id":"40","sequence_oid":null,"render_gid":"","num_q
ues":"1","header":null,"content":null,"exam_id":"80","topic_id":"0","is_linked":null,"autoHeader":"","qdata":{"40"
:{"subquestion":[{"id":"993331","group_id":"911255","sequence_id":"40","sequence_oid":null,"sequence_sid":"0"
,

"render_qid":"40","render_sqid":"","header":null,"type":"objective","category":"none","is_mco":"no","level":"me
dium","time":"120","marks":"4","neg_marks":"1","meta_id":"3390","dirty":null,"comment":null,"history":"[<b>Fe
b 05 2021 12:03:17][admin<\/b>][I][<b>admin<\/b>]","answer":"2","prefix":null,"content":"Orthoboric acid is a
very weak acid (p<sup>ka<\/sup> = 9.25). Its acidic strength increases on addition of which of the following
substance?","suffix":null,"testcases":[],"solution":null,"insert_time":"2021-02-05
12:03:17","login_id":"admin","topic_id":0,"hint":null,"qmock_list":null,"unattempt":"2","unattempt_avg_time":"1
6.67","unevaluate":"0","unevaluate_avg_time":"45","correct":"0","correct_avg_time":"45","incorrect":"0","incorr
ect_avg_time":"45","sp_answer":"","mco_marks":"0","exam_ques":"no","exam_info":null,"sub_type":null,"is_bon
us":"0","is_extra_question":"no","answer_type":null,"integer_length":null,"error_status":"0","source_q_id":"1466
988","topic":"miscellaneous","subject":"chemistry","options":["H<sub>2<\/sub>O","diols","CO&nbsp;","O<sub>2<
\/sub>
&nbsp;"]}]}}},{"id":"911256","section_id":"45865","sequence_id":"41","sequence_oid":null,"render_gid":"","num_
ques":"1","header":null,"content":null,"exam_id":"80","topic_id":"0","is_linked":null,"autoHeader":"","qdata":{"4
1":{"subquestion":[{"id":"993332","group_id":"911256","sequence_id":"41","sequence_oid":null,"sequence_sid":"
0"
,"render_qid":"41","render_sqid":"","header":null,"type":"objective","category":"none","is_mco":"no","level":"lo
w","time":"120","marks":"4","neg_marks":"1","meta_id":"3390","dirty":null,"comment":null,"history":"[<b>Feb
05 2021 12:03:17][admin<\/b>][I][<b>admin<\/b>]","answer":"2","prefix":null,"content":"The correct IUPAC name
of&nbsp;<img
src=\"https:\/\/d2lbh14zkcqlst.cloudfront.net\/content_data\/content_images\/1612266930166765500phpSBAyH
qimage.png\" style=\"vertical-align: middle;height:51px; width:150px\" \/>&nbsp;&nbsp;is
:","suffix":null,"testcases":[],"solution":null,"insert_time":"2021-02-05
12:03:17","login_id":"admin","topic_id":0,"hint":null,"qmock_list":null,"unattempt":"2","unattempt_avg_time":"1
6.33","unevaluate":"0","unevaluate_avg_time":"45","correct":"0","correct_avg_time":"45","incorrect":"0","incorr
ect_avg_time":"45","sp_answer":"","mco_marks":"0","exam_ques":"no","exam_info":null,"sub_type":null,"is_bon
us":"0","is_extra_question":"no","answer_type":null,"integer_length":null,"error_status":"0","source_q_id":"1467
004","topic":"miscellaneous","subject":"chemistry","options":["2-Methyl butanoic acid","2-Ethylprop-2-enoic
acid","2- Carboxy-1- butene","Pent-2-enoic
acid"]}]}}},{"id":"911257","section_id":"45865","sequence_id":"42","sequence_oid":null,"render_gid":"","num_qu
es":"1","header":null,"content":null,"exam_id":"80","topic_id":"0","is_linked":null,"autoHeader":"","qdata":{"42":
{"subquestion":[{"id":"993333","group_id":"911257","sequence_id":"42","sequence_oid":null,"sequence_sid":"0",

"render_qid":"42","render_sqid":"","header":null,"type":"objective","category":"none","is_mco":"no","level":"low
","time":"120","marks":"4","neg_marks":"1","meta_id":"3390","dirty":null,"comment":null,"history":"[<b>Feb 05
2021 12:03:18][admin<\/b>][I][<b>admin<\/b>]","answer":"2","prefix":null,"content":"In which of the following
compound, delocalisation of lone pair is possible?
&nbsp;","suffix":null,"testcases":[],"solution":null,"insert_time":"2021-02-05
12:03:18","login_id":"admin","topic_id":0,"hint":null,"qmock_list":null,"unattempt":"2","unattempt_avg_time":"1
6.33","unevaluate":"0","unevaluate_avg_time":"45","correct":"0","correct_avg_time":"45","incorrect":"0","incorr
ect_avg_time":"45","sp_answer":"","mco_marks":"0","exam_ques":"no","exam_info":null,"sub_type":null,"is_bon
us":"0","is_extra_question":"no","answer_type":null,"integer_length":null,"error_status":"0","source_q_id":"1467
018","topic":"miscellaneous","subject":"chemistry","options":["<img
src=\"https:\/\/d2lbh14zkcqlst.cloudfront.net\/content_data\/content_images\/1612267037857374384phpLwTzI
Rimage.png\" style=\"vertical-align: middle;height:91px; width:80px\" \/>","<img
src=\"https:\/\/d2lbh14zkcqlst.cloudfront.net\/content_data\/content_images\/1612267047394155387phpJWzv
GHimage.png\" style=\"vertical-align: middle;height:101px; width:70px\"
\/>","CH<sub>3<\/sub>CH<sub>2<\/sub>NH CH<sub>2<\/sub> CH = CH<sub>2<\/sub>","CH<sub>2<\/sub> = CH
&ndash; CH<sub>2<\/sub> &ndash;
OH"]}]}}},{"id":"911258","section_id":"45865","sequence_id":"43","sequence_oid":null,"render_gid":"","num_que
s":"1","header":null,"content":null,"exam_id":"80","topic_id":"0","is_linked":null,"autoHeader":"","qdata":{"43":{"
subquestion":[{"id":"993334","group_id":"911258","sequence_id":"43","sequence_oid":null,"sequence_sid":"0

","render_qid":"43","render_sqid":"","header":null,"type":"objective","category":"none","is_mco":"no","level":"m
edium","time":"120","marks":"4","neg_marks":"1","meta_id":"3390","dirty":null,"comment":null,"history":"[<b>F
eb 05 2021 12:03:18][admin<\/b>][I][<b>admin<\/b>]","answer":"2","prefix":null,"content":"Correct basicity order
of the following :<br \/>\n<img
src=\"https:\/\/d2lbh14zkcqlst.cloudfront.net\/content_data\/content_images\/1612267171364701211phpUq5jT
oimage.png\" style=\"vertical-align: middle;height:138px; width:500px\"
\/>","suffix":null,"testcases":[],"solution":null,"insert_time":"2021-02-05
12:03:18","login_id":"admin","topic_id":0,"hint":null,"qmock_list":null,"unattempt":"2","unattempt_avg_time":"2
0.67","unevaluate":"0","unevaluate_avg_time":"45","correct":"0","correct_avg_time":"45","incorrect":"0","incorr
ect_avg_time":"45","sp_answer":"","mco_marks":"0","exam_ques":"no","exam_info":null,"sub_type":null,"is_bon
us":"0","is_extra_question":"no","answer_type":null,"integer_length":null,"error_status":"0","source_q_id":"1467
033","topic":"miscellaneous","subject":"chemistry","options":["(II) &gt; (I) &gt; (III) &gt; (IV)","(I) &gt; (III) &gt; (II)
&gt; (IV)","(III) &gt; (I) &gt; (II) &gt; (IV)","(I) &gt; (II) &gt; (III) &gt;
(IV)"]}]}}},{"id":"911259","section_id":"45865","sequence_id":"44","sequence_oid":null,"render_gid":"","num_que
s":"1","header":null,"content":null,"exam_id":"80","topic_id":"0","is_linked":null,"autoHeader":"","qdata":{"44":{"
subquestion":[{"id":"993335","group_id":"911259","sequence_id":"44","sequence_oid":null,"sequence_sid":"0

","render_qid":"44","render_sqid":"","header":null,"type":"objective","category":"none","is_mco":"no","level":"lo
w","time":"120","marks":"4","neg_marks":"1","meta_id":"3390","dirty":null,"comment":null,"history":"[<b>Feb
05 2021 12:03:19][admin<\/b>][I][<b>admin<\/b>]","answer":"1","prefix":null,"content":"In which of the following
molecule, inductive and resonance both effects are
present?","suffix":null,"testcases":[],"solution":null,"insert_time":"2021-02-05
12:03:19","login_id":"admin","topic_id":0,"hint":null,"qmock_list":null,"unattempt":"2","unattempt_avg_time":"1
6.33","unevaluate":"0","unevaluate_avg_time":"45","correct":"0","correct_avg_time":"45","incorrect":"0","incorr
ect_avg_time":"45","sp_answer":"","mco_marks":"0","exam_ques":"no","exam_info":null,"sub_type":null,"is_bon
us":"0","is_extra_question":"no","answer_type":null,"integer_length":null,"error_status":"0","source_q_id":"1467
043","topic":"miscellaneous","subject":"chemistry","options":["<img
src=\"https:\/\/d2lbh14zkcqlst.cloudfront.net\/content_data\/content_images\/16122672561089568100phpixVlu
dimage.png\" style=\"vertical-align: middle;height:70px; width:110px\" \/>","<img
src=\"https:\/\/d2lbh14zkcqlst.cloudfront.net\/content_data\/content_images\/1612267271928952696phpFieK4
Pimage.png\" style=\"vertical-align: middle;height:79px; width:110px\" \/>","<img
src=\"https:\/\/d2lbh14zkcqlst.cloudfront.net\/content_data\/content_images\/16122672831222014060phpj24w
IPimage.png\" style=\"vertical-align: middle;height:80px; width:125px\" \/>","CH<sub>3<\/sub> &ndash;
CH<sub>3<\/sub>"]}]}}},{"id":"911260","section_id":"45865","sequence_id":"45","sequence_oid":null,"render_gid
":"","num_ques":"1","header":null,"content":null,"exam_id":"80","topic_id":"0","is_linked":null,"autoHeader":"","
qdata":{"45":{"subquestion":[{"id":"993336","group_id":"911260","sequence_id":"45","sequence_oid":null,"seque
nce_sid":"0

","render_qid":"45","render_sqid":"","header":null,"type":"objective","category":"none","is_mco":"no","level":"lo
w","time":"120","marks":"4","neg_marks":"1","meta_id":"3390","dirty":null,"comment":null,"history":"[<b>Feb
05 2021 12:03:20][admin<\/b>][I][<b>admin<\/b>]","answer":"3","prefix":null,"content":"<img
src=\"https:\/\/d2lbh14zkcqlst.cloudfront.net\/content_data\/content_images\/16122673691695452330phpAcM
Ha3image.png\" style=\"vertical-align: middle;height:126px; width:230px\" \/><br \/>\nWhich one is not a valid
resonating structure ?","suffix":null,"testcases":[],"solution":null,"insert_time":"2021-02-05
12:03:20","login_id":"admin","topic_id":0,"hint":null,"qmock_list":null,"unattempt":"2","unattempt_avg_time":"1
7","unevaluate":"0","unevaluate_avg_time":"45","correct":"0","correct_avg_time":"45","incorrect":"0","incorrect
_avg_time":"45","sp_answer":"","mco_marks":"0","exam_ques":"no","exam_info":null,"sub_type":null,"is_bonus"
:"0","is_extra_question":"no","answer_type":null,"integer_length":null,"error_status":"0","source_q_id":"146705
1","topic":"miscellaneous","subject":"chemistry","options":["I","II","III","All of
these"]}]}}},{"id":"911261","section_id":"45865","sequence_id":"46","sequence_oid":null,"render_gid":"","num_q
ues":"1","header":null,"content":null,"exam_id":"80","topic_id":"0","is_linked":null,"autoHeader":"","qdata":{"46"
:{"subquestion":[{"id":"993337","group_id":"911261","sequence_id":"46","sequence_oid":null,"sequence_sid":"0"

,"render_qid":"46","render_sqid":"","header":null,"type":"objective","category":"none","is_mco":"no","level":"me
dium","time":"120","marks":"4","neg_marks":"1","meta_id":"3390","dirty":null,"comment":null,"history":"[<b>Fe
b 05 2021 12:03:20][admin<\/b>][I][<b>admin<\/b>]","answer":"2","prefix":null,"content":"Consider the given
statements-<br \/>\n<img
src=\"https:\/\/d2lbh14zkcqlst.cloudfront.net\/content_data\/content_images\/1612267474824934658phpk8mC
nyimage.png\" style=\"vertical-align: middle;height:267px; width:500px\" \/><br \/>\nWhich one is correct
option?<br \/>\n&nbsp;","suffix":null,"testcases":[],"solution":null,"insert_time":"2021-02-05
12:03:20","login_id":"admin","topic_id":0,"hint":null,"qmock_list":null,"unattempt":"2","unattempt_avg_time":"2
1","unevaluate":"0","unevaluate_avg_time":"45","correct":"0","correct_avg_time":"45","incorrect":"0","incorrect
_avg_time":"45","sp_answer":"","mco_marks":"0","exam_ques":"no","exam_info":null,"sub_type":null,"is_bonus"
:"0","is_extra_question":"no","answer_type":null,"integer_length":null,"error_status":"0","source_q_id":"146706
2","topic":"miscellaneous","subject":"chemistry","options":["I, II and III","II, III and IV","I, II and IV&nbsp;","I, III
and
IV"]}]}}},{"id":"911262","section_id":"45865","sequence_id":"47","sequence_oid":null,"render_gid":"","num_ques
":"1","header":null,"content":null,"exam_id":"80","topic_id":"0","is_linked":null,"autoHeader":"","qdata":{"47":{"s
ubquestion":[{"id":"993338","group_id":"911262","sequence_id":"47","sequence_oid":null,"sequence_sid":"0

","render_qid":"47","render_sqid":"","header":null,"type":"objective","category":"none","is_mco":"no","level":"m
edium","time":"120","marks":"4","neg_marks":"1","meta_id":"3390","dirty":null,"comment":null,"history":"[<b>F
eb 05 2021 12:03:20][admin<\/b>][I][<b>admin<\/b>]","answer":"1","prefix":null,"content":"Correct order of
acidic strength is :<br \/>\n<img
src=\"https:\/\/d2lbh14zkcqlst.cloudfront.net\/content_data\/content_images\/1612267570561452056phpECVO
Uiimage.png\" style=\"vertical-align: middle;height:112px; width:550px\"
\/>","suffix":null,"testcases":[],"solution":null,"insert_time":"2021-02-05
12:03:20","login_id":"admin","topic_id":0,"hint":null,"qmock_list":null,"unattempt":"2","unattempt_avg_time":"2
0","unevaluate":"0","unevaluate_avg_time":"45","correct":"0","correct_avg_time":"45","incorrect":"0","incorrect
_avg_time":"45","sp_answer":"","mco_marks":"0","exam_ques":"no","exam_info":null,"sub_type":null,"is_bonus"
:"0","is_extra_question":"no","answer_type":null,"integer_length":null,"error_status":"0","source_q_id":"146707
4","topic":"miscellaneous","subject":"chemistry","options":["II &gt; I &gt; III &gt; IV&nbsp;","I &gt; II &gt; III &gt;
IV","III &gt; II &gt; I &gt; IV","IV &gt; I &gt; II &gt;
III"]}]}}},{"id":"911263","section_id":"45865","sequence_id":"48","sequence_oid":null,"render_gid":"","num_ques"
:"1","header":null,"content":null,"exam_id":"80","topic_id":"0","is_linked":null,"autoHeader":"","qdata":{"48":{"s
ubquestion":[{"id":"993339","group_id":"911263","sequence_id":"48","sequence_oid":null,"sequence_sid":"0
","render_qid":"48","render_sqid":"","header":null,"type":"objective","category":"none","is_mco":"no","level":"lo
w","time":"120","marks":"4","neg_marks":"1","meta_id":"3390","dirty":null,"comment":null,"history":"[<b>Feb
05 2021 12:03:21][admin<\/b>][I][<b>admin<\/b>]","answer":"2","prefix":null,"content":"Correct order of basic
strength is :&nbsp;&nbsp; &nbsp;<br \/>\n<img
src=\"https:\/\/d2lbh14zkcqlst.cloudfront.net\/content_data\/content_images\/1612267699911710844phpqc9xw
Dimage.png\" style=\"vertical-align: middle;height:165px; width:450px\"
\/>","suffix":null,"testcases":[],"solution":null,"insert_time":"2021-02-05
12:03:21","login_id":"admin","topic_id":0,"hint":null,"qmock_list":null,"unattempt":"2","unattempt_avg_time":"4
2.33","unevaluate":"0","unevaluate_avg_time":"45","correct":"0","correct_avg_time":"45","incorrect":"0","incorr
ect_avg_time":"45","sp_answer":"","mco_marks":"0","exam_ques":"no","exam_info":null,"sub_type":null,"is_bon
us":"0","is_extra_question":"no","answer_type":null,"integer_length":null,"error_status":"0","source_q_id":"1467
088","topic":"miscellaneous","subject":"chemistry","options":["I &gt; II &gt; III &gt; IV","IV &gt; III &gt; II
&nbsp;&gt; I","I &gt; II &gt; III &gt; IV","I &gt; II &gt; III &gt;
IV"]}]}}},{"id":"911264","section_id":"45865","sequence_id":"49","sequence_oid":null,"render_gid":"","num_ques
":"1","header":null,"content":null,"exam_id":"80","topic_id":"0","is_linked":null,"autoHeader":"","qdata":{"49":{"s
ubquestion":[{"id":"993340","group_id":"911264","sequence_id":"49","sequence_oid":null,"sequence_sid":"0"

,"render_qid":"49","render_sqid":"","header":null,"type":"objective","category":"none","is_mco":"no","level":"lo
w","time":"120","marks":"4","neg_marks":"1","meta_id":"3390","dirty":null,"comment":null,"history":"[<b>Feb
05 2021 12:03:22][admin<\/b>][I][<b>admin<\/b>]","answer":"2","prefix":null,"content":"Most stable resonating
structure of diazomethane CH<sub>2<\/sub>N<sub>2<\/sub>
&nbsp;is","suffix":null,"testcases":[],"solution":null,"insert_time":"2021-02-05
12:03:22","login_id":"admin","topic_id":0,"hint":null,"qmock_list":null,"unattempt":"2","unattempt_avg_time":"2
6.33","unevaluate":"0","unevaluate_avg_time":"45","correct":"0","correct_avg_time":"45","incorrect":"0","incorr
ect_avg_time":"45","sp_answer":"","mco_marks":"0","exam_ques":"no","exam_info":null,"sub_type":null,"is_bon
us":"0","is_extra_question":"no","answer_type":null,"integer_length":null,"error_status":"0","source_q_id":"1467
096","topic":"miscellaneous","subject":"chemistry","options":["<img
src=\"https:\/\/d2lbh14zkcqlst.cloudfront.net\/content_data\/content_images\/1612267790431990729phpOVzG
RWimage.png\" style=\"vertical-align: middle;height:39px; width:100px\" \/>","<img
src=\"https:\/\/d2lbh14zkcqlst.cloudfront.net\/content_data\/content_images\/1612267804849228023phpC5NDl
Oimage.png\" style=\"vertical-align: middle;height:42px; width:100px\" \/>","<img
src=\"https:\/\/d2lbh14zkcqlst.cloudfront.net\/content_data\/content_images\/1612267819788889152phpejTSr1
image.png\" style=\"vertical-align: middle;height:33px; width:100px\" \/>","<img
src=\"https:\/\/d2lbh14zkcqlst.cloudfront.net\/content_data\/content_images\/16122678302091248832phphwoJ
6Ximage.png\" style=\"vertical-align: middle;height:34px; width:100px\"
\/>"]}]}}},{"id":"911265","section_id":"45865","sequence_id":"50","sequence_oid":null,"render_gid":"","num_que
s":"1","header":null,"content":null,"exam_id":"80","topic_id":"0","is_linked":null,"autoHeader":"","qdata":{"50":{"
subquestion":[{"id":"993341","group_id":"911265","sequence_id":"50","sequence_oid":null,"sequence_sid":"0
","render_qid":"50","render_sqid":"","header":null,"type":"objective","category":"none","is_mco":"no","level":"lo
w","time":"120","marks":"4","neg_marks":"1","meta_id":"3390","dirty":null,"comment":null,"history":"[<b>Feb
05 2021 12:03:23][admin<\/b>][I][<b>admin<\/b>]","answer":"3","prefix":null,"content":"Which of the following
compounds have only one type of hybridisation of the carbon atom-<br \/>\n<img
src=\"https:\/\/d2lbh14zkcqlst.cloudfront.net\/content_data\/content_images\/1612267895613794383phpCY6RC
Fimage.png\" style=\"vertical-align: middle;height:137px; width:250px\"
\/>","suffix":null,"testcases":[],"solution":null,"insert_time":"2021-02-05
12:03:23","login_id":"admin","topic_id":0,"hint":null,"qmock_list":null,"unattempt":"2","unattempt_avg_time":"1
8.67","unevaluate":"0","unevaluate_avg_time":"45","correct":"0","correct_avg_time":"45","incorrect":"0","incorr
ect_avg_time":"45","sp_answer":"","mco_marks":"0","exam_ques":"no","exam_info":null,"sub_type":null,"is_bon
us":"0","is_extra_question":"no","answer_type":null,"integer_length":null,"error_status":"0","source_q_id":"1467
103","topic":"miscellaneous","subject":"chemistry","options":["a,b","b, c","a, b,
d","All"]}]}}}]},{"id":"45866","paper_id":"12803","section":"Chemistry","subsection":"SEC-
B","sequence_id":"51","sequence_sid":"0","header":"<div>\r\n<ul>\r\n\t<li>This section has <b>TEN<\/b>
questions.<\/li>\r\n\t<li>Candidates have to <b>attempt any 5 questions out of 10<\/b>. If more than 5 questions
are attempted, then only first 5 attempted questions will be evaluated.<\/li>\r\n\t<li>The answer to each question
is a <b>NUMERICAL VALUE<\/b>.<\/li>\r\n\t<li>For each question, enter the correct numerical value (in decimal
notation, truncated\/rounded-off to the <b>second decimal place <\/b>e.g. 6.25, 7.00, &ndash;0.33, &ndash;.30,
30.27, &ndash;127.30, if answer is 11.36777..... then both 11.36 and 11.37 will be correct).<\/li>\r\n\t<li>For each
question, type the correct integer in the space provided below the question using provided number
keys.<\/li>\r\n\t<li><b>Marking scheme:<\/b>\r\n\t<ul>\r\n\t\t<li>Full Marks : +4 If the correct integer is typed in
the provided space.<\/li>\r\n\t\t<li>Zero Marks : 0 In all other
cases.<\/li>\r\n\t<\/ul>\r\n\t<\/li>\r\n<\/ul>\r\n<\/div>\r\n","num_grps":"10","marks":"20","neg_marks":"0","ti
me_duration":"20","type":"paper","ans_limit":"5","gdata":[{"id":"911266","section_id":"45866","sequence_id":"5
1","sequence_oid":null,"render_gid":"","num_ques":"1","header":null,"content":null,"exam_id":"80","topic_id":"0
","is_linked":null,"autoHeader":"","qdata":{"51":{"subquestion":[{"id":"993342","group_id":"911266","sequence_i
d":"51","sequence_oid":null,"sequence_sid":"0

","render_qid":"51","render_sqid":"","header":null,"type":"subjective","category":"none","is_mco":"no","level":"h
igh","time":"120","marks":"4","neg_marks":"0","meta_id":"3390","dirty":null,"comment":null,"history":"[<b>Feb
05 2021 12:03:23][admin<\/b>][I][<b>admin<\/b>]","answer":"4.00","prefix":null,"content":"Identify the number
of B atoms in colemanite [Ca<sub>2<\/sub>B<sub>6<\/sub>O<sub>11<\/sub>.5H<sub>2<\/sub>O] with no back
bonding.","suffix":null,"testcases":[],"solution":null,"insert_time":"2021-02-05
12:03:23","login_id":"admin","topic_id":0,"hint":null,"qmock_list":null,"unattempt":"0","unattempt_avg_time":"4
5","unevaluate":"0","unevaluate_avg_time":"45","correct":"2","correct_avg_time":"20","incorrect":"0","incorrect
_avg_time":"45","sp_answer":"","mco_marks":"0","exam_ques":"no","exam_info":null,"sub_type":"numerical","is
_bonus":"0","is_extra_question":"no","answer_type":null,"integer_length":null,"error_status":"0","source_q_id":"
1467108","topic":"miscellaneous","subject":"chemistry"}]}}},{"id":"911267","section_id":"45866","sequence_id":"
52","sequence_oid":null,"render_gid":"","num_ques":"1","header":null,"content":null,"exam_id":"80","topic_id":"
0","is_linked":null,"autoHeader":"","qdata":{"52":{"subquestion":[{"id":"993343","group_id":"911267","sequence
_id":"52","sequence_oid":null,"sequence_sid":"0
","render_qid":"52","render_sqid":"","header":null,"type":"subjective","category":"none","is_mco":"no","level":"
medium","time":"120","marks":"4","neg_marks":"0","meta_id":"3390","dirty":null,"comment":null,"history":"[<b>
Feb 05 2021 12:03:23][admin<\/b>][I][<b>admin<\/b>]","answer":"6.00","prefix":null,"content":"Find out the
number of identical Al &ndash; Cl bonds around Al in AlCl<sub>3<\/sub>(solid) -
","suffix":null,"testcases":[],"solution":null,"insert_time":"2021-02-05
12:03:23","login_id":"admin","topic_id":0,"hint":null,"qmock_list":null,"unattempt":"2","unattempt_avg_time":"1
6.33","unevaluate":"0","unevaluate_avg_time":"45","correct":"0","correct_avg_time":"45","incorrect":"0","incorr
ect_avg_time":"45","sp_answer":"","mco_marks":"0","exam_ques":"no","exam_info":null,"sub_type":"numerical"
,"is_bonus":"0","is_extra_question":"no","answer_type":null,"integer_length":null,"error_status":"0","source_q_i
d":"1467114","topic":"miscellaneous","subject":"chemistry"}]}}},{"id":"911268","section_id":"45866","sequence_i
d":"53","sequence_oid":null,"render_gid":"","num_ques":"1","header":null,"content":null,"exam_id":"80","topic_i
d":"0","is_linked":null,"autoHeader":"","qdata":{"53":{"subquestion":[{"id":"993344","group_id":"911268","seque
nce_id":"53","sequence_oid":null,"sequence_sid":"0

","render_qid":"53","render_sqid":"","header":null,"type":"subjective","category":"none","is_mco":"no","level":"
medium","time":"120","marks":"4","neg_marks":"0","meta_id":"3390","dirty":null,"comment":null,"history":"[<b>
Feb 05 2021 12:03:23][admin<\/b>][I][<b>admin<\/b>]","answer":"6.00","prefix":null,"content":"The number of
moles of H-bonds formed by 1 mole of ortho-boric in it&rsquo;s crystal form-
&nbsp;","suffix":null,"testcases":[],"solution":null,"insert_time":"2021-02-05
12:03:23","login_id":"admin","topic_id":0,"hint":null,"qmock_list":null,"unattempt":"2","unattempt_avg_time":"1
5.67","unevaluate":"0","unevaluate_avg_time":"45","correct":"0","correct_avg_time":"45","incorrect":"0","incorr
ect_avg_time":"45","sp_answer":"","mco_marks":"0","exam_ques":"no","exam_info":null,"sub_type":"numerical"
,"is_bonus":"0","is_extra_question":"no","answer_type":null,"integer_length":null,"error_status":"0","source_q_i
d":"1467118","topic":"miscellaneous","subject":"chemistry"}]}}},{"id":"911269","section_id":"45866","sequence_i
d":"54","sequence_oid":null,"render_gid":"","num_ques":"1","header":null,"content":null,"exam_id":"80","topic_i
d":"0","is_linked":null,"autoHeader":"","qdata":{"54":{"subquestion":[{"id":"993345","group_id":"911269","seque
nce_id":"54","sequence_oid":null,"sequence_sid":"0

","render_qid":"54","render_sqid":"","header":null,"type":"subjective","category":"none","is_mco":"no","level":"
medium","time":"120","marks":"4","neg_marks":"0","meta_id":"3390","dirty":null,"comment":null,"history":"[<b>
Feb 05 2021 12:03:23][admin<\/b>][I][<b>admin<\/b>]","answer":"3.00","prefix":null,"content":"Identify the
number of atoms present in same plane in gaseous H<sub>2<\/sub>O<sub>2<\/sub> &nbsp;&nbsp;
&nbsp;","suffix":null,"testcases":[],"solution":null,"insert_time":"2021-02-05
12:03:23","login_id":"admin","topic_id":0,"hint":null,"qmock_list":null,"unattempt":"2","unattempt_avg_time":"1
6","unevaluate":"0","unevaluate_avg_time":"45","correct":"0","correct_avg_time":"45","incorrect":"0","incorrect
_avg_time":"45","sp_answer":"","mco_marks":"0","exam_ques":"no","exam_info":null,"sub_type":"numerical","is
_bonus":"0","is_extra_question":"no","answer_type":null,"integer_length":null,"error_status":"0","source_q_id":"
1467120","topic":"miscellaneous","subject":"chemistry"}]}}},{"id":"911270","section_id":"45866","sequence_id":"
55","sequence_oid":null,"render_gid":"","num_ques":"1","header":null,"content":null,"exam_id":"80","topic_id":"
0","is_linked":null,"autoHeader":"","qdata":{"55":{"subquestion":[{"id":"993346","group_id":"911270","sequence
_id":"55","sequence_oid":null,"sequence_sid":"0
","render_qid":"55","render_sqid":"","header":null,"type":"subjective","category":"none","is_mco":"no","level":"
medium","time":"120","marks":"4","neg_marks":"0","meta_id":"3390","dirty":null,"comment":null,"history":"[<b>
Feb 05 2021 12:03:23][admin<\/b>][I][<b>admin<\/b>]","answer":"4.00","prefix":null,"content":"Total how many
moles of acids are obtained on complete hydrolysis of 1 mole
BCl<sub>3<\/sub>?","suffix":null,"testcases":[],"solution":null,"insert_time":"2021-02-05
12:03:23","login_id":"admin","topic_id":0,"hint":null,"qmock_list":null,"unattempt":"2","unattempt_avg_time":"1
6","unevaluate":"0","unevaluate_avg_time":"45","correct":"0","correct_avg_time":"45","incorrect":"0","incorrect
_avg_time":"45","sp_answer":"","mco_marks":"0","exam_ques":"no","exam_info":null,"sub_type":"numerical","is
_bonus":"0","is_extra_question":"no","answer_type":null,"integer_length":null,"error_status":"0","source_q_id":"
1467122","topic":"miscellaneous","subject":"chemistry"}]}}},{"id":"911271","section_id":"45866","sequence_id":"
56","sequence_oid":null,"render_gid":"","num_ques":"1","header":null,"content":null,"exam_id":"80","topic_id":"
0","is_linked":null,"autoHeader":"","qdata":{"56":{"subquestion":[{"id":"993347","group_id":"911271","sequence
_id":"56","sequence_oid":null,"sequence_sid":"0","

render_qid":"56","render_sqid":"","header":null,"type":"subjective","category":"none","is_mco":"no","level":"low
","time":"120","marks":"4","neg_marks":"0","meta_id":"3390","dirty":null,"comment":null,"history":"[<b>Feb 05
2021 12:03:23][admin<\/b>][I][<b>admin<\/b>]","answer":"2.00","prefix":null,"content":"<img
src=\"https:\/\/d2lbh14zkcqlst.cloudfront.net\/content_data\/content_images\/16122681621679754919php0QG
V6fimage.png\" style=\"vertical-align: middle;height:97px; width:220px\" \/><br \/>\nTotal number of delocalised
lone pair in given compound is\/are.","suffix":null,"testcases":[],"solution":null,"insert_time":"2021-02-05
12:03:23","login_id":"admin","topic_id":0,"hint":null,"qmock_list":null,"unattempt":"2","unattempt_avg_time":"1
6.67","unevaluate":"0","unevaluate_avg_time":"45","correct":"0","correct_avg_time":"45","incorrect":"0","incorr
ect_avg_time":"45","sp_answer":"","mco_marks":"0","exam_ques":"no","exam_info":null,"sub_type":"numerical"
,"is_bonus":"0","is_extra_question":"no","answer_type":null,"integer_length":null,"error_status":"0","source_q_i
d":"1467128","topic":"miscellaneous","subject":"chemistry"}]}}},{"id":"911272","section_id":"45866","sequence_i
d":"57","sequence_oid":null,"render_gid":"","num_ques":"1","header":null,"content":null,"exam_id":"80","topic_i
d":"0","is_linked":null,"autoHeader":"","qdata":{"57":{"subquestion":[{"id":"993348","group_id":"911272","seque
nce_id":"57","sequence_oid":null,"sequence_sid":"0

","render_qid":"57","render_sqid":"","header":null,"type":"subjective","category":"none","is_mco":"no","level":"l
ow","time":"120","marks":"4","neg_marks":"0","meta_id":"3390","dirty":null,"comment":null,"history":"[<b>Feb
05 2021 12:03:23][admin<\/b>][I][<b>admin<\/b>]","answer":"6.00","prefix":null,"content":"What is the ratio of
number of &sigma;- and &pi;-bonds in the molecule&nbsp;<img
src=\"https:\/\/d2lbh14zkcqlst.cloudfront.net\/content_data\/content_images\/1612268237175403884phpIafZTji
mage.png\" style=\"vertical-align: middle;height:49px; width:160px\"
\/>","suffix":null,"testcases":[],"solution":null,"insert_time":"2021-02-05
12:03:23","login_id":"admin","topic_id":0,"hint":null,"qmock_list":null,"unattempt":"2","unattempt_avg_time":"1
6.33","unevaluate":"0","unevaluate_avg_time":"45","correct":"0","correct_avg_time":"45","incorrect":"0","incorr
ect_avg_time":"45","sp_answer":"","mco_marks":"0","exam_ques":"no","exam_info":null,"sub_type":"numerical"
,"is_bonus":"0","is_extra_question":"no","answer_type":null,"integer_length":null,"error_status":"0","source_q_i
d":"1467132","topic":"miscellaneous","subject":"chemistry"}]}}},{"id":"911273","section_id":"45866","sequence_i
d":"58","sequence_oid":null,"render_gid":"","num_ques":"1","header":null,"content":null,"exam_id":"80","topic_i
d":"0","is_linked":null,"autoHeader":"","qdata":{"58":{"subquestion":[{"id":"993349","group_id":"911273","seque
nce_id":"58","sequence_oid":null,"sequence_sid":"0"

,"render_qid":"58","render_sqid":"","header":null,"type":"subjective","category":"none","is_mco":"no","level":"hi
gh","time":"120","marks":"4","neg_marks":"0","meta_id":"3390","dirty":null,"comment":null,"history":"[<b>Feb
05 2021 12:03:24][admin<\/b>][I][<b>admin<\/b>]","answer":"6.00","prefix":null,"content":"How many of the
following compounds have even number of 2&deg; carbon atoms.<br \/>\n<img
src=\"https:\/\/d2lbh14zkcqlst.cloudfront.net\/content_data\/content_images\/16122683291193901965phpJOw
7R4image.png\" style=\"vertical-align: middle;height:291px; width:600px\"
\/>","suffix":null,"testcases":[],"solution":null,"insert_time":"2021-02-05
12:03:24","login_id":"admin","topic_id":0,"hint":null,"qmock_list":null,"unattempt":"2","unattempt_avg_time":"2
0.33","unevaluate":"0","unevaluate_avg_time":"45","correct":"0","correct_avg_time":"45","incorrect":"0","incorr
ect_avg_time":"45","sp_answer":"","mco_marks":"0","exam_ques":"no","exam_info":null,"sub_type":"numerical"
,"is_bonus":"0","is_extra_question":"no","answer_type":null,"integer_length":null,"error_status":"0","source_q_i
d":"1467138","topic":"miscellaneous","subject":"chemistry"}]}}},{"id":"911274","section_id":"45866","sequence_i
d":"59","sequence_oid":null,"render_gid":"","num_ques":"1","header":null,"content":null,"exam_id":"80","topic_i
d":"0","is_linked":null,"autoHeader":"","qdata":{"59":{"subquestion":[{"id":"993350","group_id":"911274","seque
nce_id":"59","sequence_oid":null,"sequence_sid":"0

","render_qid":"59","render_sqid":"","header":null,"type":"subjective","category":"none","is_mco":"no","level":"l
ow","time":"120","marks":"4","neg_marks":"0","meta_id":"3390","dirty":null,"comment":null,"history":"[<b>Feb
05 2021 12:03:24][admin<\/b>][I][<b>admin<\/b>]","answer":"3.00","prefix":null,"content":"Total number of
substituents on principal chain in given compound :\n<div style=\"text-align:center\"><img
src=\"https:\/\/d2lbh14zkcqlst.cloudfront.net\/content_data\/content_images\/161226839496200042phpmOcTc
Aimage.png\" style=\"vertical-align: middle;height:76px; width:150px\"
\/><\/div>\n","suffix":null,"testcases":[],"solution":null,"insert_time":"2021-02-05
12:03:24","login_id":"admin","topic_id":0,"hint":null,"qmock_list":null,"unattempt":"2","unattempt_avg_time":"1
6","unevaluate":"0","unevaluate_avg_time":"45","correct":"0","correct_avg_time":"45","incorrect":"0","incorrect
_avg_time":"45","sp_answer":"","mco_marks":"0","exam_ques":"no","exam_info":null,"sub_type":"numerical","is
_bonus":"0","is_extra_question":"no","answer_type":null,"integer_length":null,"error_status":"0","source_q_id":"
1467139","topic":"miscellaneous","subject":"chemistry"}]}}},{"id":"911275","section_id":"45866","sequence_id":"
60","sequence_oid":null,"render_gid":"","num_ques":"1","header":null,"content":null,"exam_id":"80","topic_id":"
0","is_linked":null,"autoHeader":"","qdata":{"60":{"subquestion":[{"id":"993351","group_id":"911275","sequence
_id":"60","sequence_oid":null,"sequence_sid":"0","
render_qid":"60","render_sqid":"","header":null,"type":"subjective","category":"none","is_mco":"no","level":"low
","time":"120","marks":"4","neg_marks":"0","meta_id":"3390","dirty":null,"comment":null,"history":"[<b>Feb 05
2021 12:03:24][admin<\/b>][I][<b>admin<\/b>]","answer":"3.00","prefix":null,"content":"In how many of the
following direction of &nbsp;inductive effect is correctly shown.<br \/>\n<img
src=\"https:\/\/d2lbh14zkcqlst.cloudfront.net\/content_data\/content_images\/1612268466633583052phpWsU
W1Rimage.png\" style=\"vertical-align: middle;height:293px; width:550px\"
\/>","suffix":null,"testcases":[],"solution":null,"insert_time":"2021-02-05
12:03:24","login_id":"admin","topic_id":0,"hint":null,"qmock_list":null,"unattempt":"2","unattempt_avg_time":"1
9","unevaluate":"0","unevaluate_avg_time":"45","correct":"0","correct_avg_time":"45","incorrect":"0","incorrect
_avg_time":"45","sp_answer":"","mco_marks":"0","exam_ques":"no","exam_info":null,"sub_type":"numerical","is
_bonus":"0","is_extra_question":"no","answer_type":null,"integer_length":null,"error_status":"0","source_q_id":"
1467144","topic":"miscellaneous","subject":"chemistry"}]}}}]}]},"Mathematics":{"subsections":[{"id":"45867","pap
er_id":"12803","section":"Mathematics","subsection":"SEC-
A","sequence_id":"61","sequence_sid":"0","header":"<ul>\r\n\t<li>This section contains <b>TWENTY<\/b>
questions.<\/li>\r\n\t<li>Each question has FOUR options (A), (B), (C) and (D). <b>ONLY ONE<\/b> of these four
options is correct.<\/li>\r\n\t<li>For each question, select the alphabet corresponding to the correct option
provided below the questions.<\/li>\r\n\t<li><b>Marking scheme:<\/b>&nbsp;\r\n\t<ul>\r\n\t\t<li>Full Marks :
<span style=\"color:black\">+4 If correct answer is selected.<\/span><\/li>\r\n\t\t<li><span
style=\"color:black\">Zero Marks <\/span>: <span style=\"color:black\">0&nbsp;If none of the option is
selected.<\/span><\/li>\r\n\t\t<li><span style=\"color:black\">Negative Marks <\/span>: <span
style=\"color:black\">&ndash;1 If wrong option is
selected.<\/span><\/li>\r\n\t<\/ul>\r\n\t<\/li>\r\n<\/ul>\r\n","num_grps":"20","marks":"80","neg_marks":"0","ti
me_duration":"40","type":"paper","ans_limit":"0","gdata":[{"id":"911276","section_id":"45867","sequence_id":"6
1","sequence_oid":null,"render_gid":"","num_ques":"1","header":null,"content":null,"exam_id":"80","topic_id":"0
","is_linked":null,"autoHeader":"","qdata":{"61":{"subquestion":[{"id":"993352","group_id":"911276","sequence_i
d":"61","sequence_oid":null,"sequence_sid":"0

,"render_qid":"61","render_sqid":"","header":null,"type":"objective","category":"none","is_mco":"no","level":"lo
w","time":"120","marks":"4","neg_marks":"1","meta_id":"3420","dirty":null,"comment":null,"history":"[<b>Feb
05 2021 12:03:26][admin<\/b>][I][<b>admin<\/b>]","answer":"1","prefix":null,"content":"In a triangle ABC
&nbsp;the value of&nbsp;<img alt=\"frac{{cos A}}{a} + frac{{cos B}}{b} + frac{{cos C}}{c}\"
src=\"https:\/\/d2lbh14zkcqlst.cloudfront.net\/content_data\/content_images\/16123289211031250103.png\"
\/>&nbsp;&nbsp;is (where a,b,c are the length of side BC, CA,
AB,)","suffix":null,"testcases":[],"solution":null,"insert_time":"2021-02-05
12:03:26","login_id":"admin","topic_id":0,"hint":null,"qmock_list":null,"unattempt":"0","unattempt_avg_time":"4
5","unevaluate":"0","unevaluate_avg_time":"45","correct":"2","correct_avg_time":"32","incorrect":"0","incorrect
_avg_time":"45","sp_answer":"","mco_marks":"0","exam_ques":"no","exam_info":null,"sub_type":null,"is_bonus"
:"0","is_extra_question":"no","answer_type":null,"integer_length":null,"error_status":"0","source_q_id":"146754
6","topic":"miscellaneous","subject":"mathematics","options":["<img alt=\"frac{{{a^2} + {b^2} + {c^2}}}{{2abc}}\"
src=\"https:\/\/d2lbh14zkcqlst.cloudfront.net\/content_data\/content_images\/16123289321335512418.png\"
\/>","<img alt=\"frac{{{a^2} + {b^2} + {c^2}}}{{abc}}\"
src=\"https:\/\/d2lbh14zkcqlst.cloudfront.net\/content_data\/content_images\/16123289381420129504.png\"
\/>","<img alt=\"frac{{{a^2} + {b^2} + {c^2}}}{{3abc}}\"
src=\"https:\/\/d2lbh14zkcqlst.cloudfront.net\/content_data\/content_images\/1612328944467474376.png\"
\/>","<img alt=\"frac{{{a^2} + {b^2} - {c^2}}}{{abc}}\"
src=\"https:\/\/d2lbh14zkcqlst.cloudfront.net\/content_data\/content_images\/161232895596313212.png\"
\/>"]}]}}},{"id":"911277","section_id":"45867","sequence_id":"62","sequence_oid":null,"render_gid":"","num_que
s":"1","header":null,"content":null,"exam_id":"80","topic_id":"0","is_linked":null,"autoHeader":"","qdata":{"62":{"
subquestion":[{"id":"993353","group_id":"911277","sequence_id":"62","sequence_oid":null,"sequence_sid":"0

","render_qid":"62","render_sqid":"","header":null,"type":"objective","category":"none","is_mco":"no","level":"lo
w","time":"120","marks":"4","neg_marks":"1","meta_id":"3420","dirty":null,"comment":null,"history":"[<b>Feb
05 2021 12:03:27][admin<\/b>][I][<b>admin<\/b>]","answer":"2","prefix":null,"content":"If &nbsp;a, b, c
&nbsp;are in &nbsp;A.P. &nbsp;then the numerical value of &nbsp;<img
src=\"https:\/\/d2lbh14zkcqlst.cloudfront.net\/content_data\/content_images\/16123290121750565041phpMwa
Eo1image.png\" style=\"vertical-align: middle;height:56px; width:90px\" \/>&nbsp;is
:","suffix":null,"testcases":[],"solution":null,"insert_time":"2021-02-05
12:03:27","login_id":"admin","topic_id":0,"hint":null,"qmock_list":null,"unattempt":"2","unattempt_avg_time":"1
9.33","unevaluate":"0","unevaluate_avg_time":"45","correct":"0","correct_avg_time":"45","incorrect":"0","incorr
ect_avg_time":"45","sp_answer":"","mco_marks":"0","exam_ques":"no","exam_info":null,"sub_type":null,"is_bon
us":"0","is_extra_question":"no","answer_type":null,"integer_length":null,"error_status":"0","source_q_id":"1467
566","topic":"miscellaneous","subject":"mathematics","options":["1\/4","1\/3","1\/2","1"]}]}}},{"id":"911278","sec
tion_id":"45867","sequence_id":"63","sequence_oid":null,"render_gid":"","num_ques":"1","header":null,"content
":null,"exam_id":"80","topic_id":"0","is_linked":null,"autoHeader":"","qdata":{"63":{"subquestion":[{"id":"993354"
,"group_id":"911278","sequence_id":"63","sequence_oid":null,"sequence_sid":"

,"render_qid":"63","render_sqid":"","header":null,"type":"objective","category":"none","is_mco":"no","level":"lo
w","time":"120","marks":"4","neg_marks":"1","meta_id":"3420","dirty":null,"comment":null,"history":"[<b>Feb
05 2021 12:03:29][admin<\/b>][I][<b>admin<\/b>]","answer":"1","prefix":null,"content":"If p<sub>1<\/sub>,
p<sub>2<\/sub>, p<sub>3<\/sub> are respectively the perpendicular from the vertices of a triangle on the sides ,
then the value of &nbsp;<img alt=\"frac{1}{{{p_1}}}\"
src=\"https:\/\/d2lbh14zkcqlst.cloudfront.net\/content_data\/content_images\/1612329182462450007.png\"
\/>&nbsp;+&nbsp;<img alt=\"frac{1}{{{p_2}}}\"
src=\"https:\/\/d2lbh14zkcqlst.cloudfront.net\/content_data\/content_images\/16123291911641940829.png\"
\/>&nbsp;&nbsp;+&nbsp;<img alt=\"frac{1}{{{p_3}}}\"
src=\"https:\/\/d2lbh14zkcqlst.cloudfront.net\/content_data\/content_images\/1612329200920162793.png\"
\/>.","suffix":null,"testcases":[],"solution":null,"insert_time":"2021-02-05
12:03:29","login_id":"admin","topic_id":0,"hint":null,"qmock_list":null,"unattempt":"2","unattempt_avg_time":"1
7.67","unevaluate":"0","unevaluate_avg_time":"45","correct":"0","correct_avg_time":"45","incorrect":"0","incorr
ect_avg_time":"45","sp_answer":"","mco_marks":"0","exam_ques":"no","exam_info":null,"sub_type":null,"is_bon
us":"0","is_extra_question":"no","answer_type":null,"integer_length":null,"error_status":"0","source_q_id":"1467
577","topic":"miscellaneous","subject":"mathematics","options":["<img alt=\"frac{1}{r}\"
src=\"https:\/\/d2lbh14zkcqlst.cloudfront.net\/content_data\/content_images\/16123292222078577298.png\"
\/>","<img alt=\"frac{1}{R}\"
src=\"https:\/\/d2lbh14zkcqlst.cloudfront.net\/content_data\/content_images\/1612329228207923049.png\"
\/>","<img alt=\"frac{1}{Delta }\"
src=\"https:\/\/d2lbh14zkcqlst.cloudfront.net\/content_data\/content_images\/16123292331958032938.png\"
\/>","r"]}]}}},{"id":"911279","section_id":"45867","sequence_id":"64","sequence_oid":null,"render_gid":"","num_
ques":"1","header":null,"content":null,"exam_id":"80","topic_id":"0","is_linked":null,"autoHeader":"","qdata":{"6
4":{"subquestion":[{"id":"993355","group_id":"911279","sequence_id":"64","sequence_oid":null,"sequence_sid":"
0

","render_qid":"64","render_sqid":"","header":null,"type":"objective","category":"none","is_mco":"no","level":"m
edium","time":"120","marks":"4","neg_marks":"1","meta_id":"3420","dirty":null,"comment":null,"history":"[<b>F
eb 05 2021 12:03:30][admin<\/b>][I][<b>admin<\/b>]","answer":"1","prefix":null,"content":"The range of the
function f(x) =&nbsp;<img alt=\"frac{{{x^2} - 3x + 2}}{{{x^2} - 1}}\"
src=\"https:\/\/d2lbh14zkcqlst.cloudfront.net\/content_data\/content_images\/16123292871812407550.png\"
\/>&nbsp;&nbsp;is&nbsp;&nbsp; &nbsp;","suffix":null,"testcases":[],"solution":null,"insert_time":"2021-02-05
12:03:30","login_id":"admin","topic_id":0,"hint":null,"qmock_list":null,"unattempt":"2","unattempt_avg_time":"1
7","unevaluate":"0","unevaluate_avg_time":"45","correct":"0","correct_avg_time":"45","incorrect":"0","incorrect
_avg_time":"45","sp_answer":"","mco_marks":"0","exam_ques":"no","exam_info":null,"sub_type":null,"is_bonus"
:"0","is_extra_question":"no","answer_type":null,"integer_length":null,"error_status":"0","source_q_id":"146758
3","topic":"miscellaneous","subject":"mathematics","options":["<img alt=\"R - left{ { - frac{1}{2},1} right}\"
src=\"https:\/\/d2lbh14zkcqlst.cloudfront.net\/content_data\/content_images\/16123292961899045994.png\"
\/>","<img alt=\"R - left{ { - 1} right}\"
src=\"https:\/\/d2lbh14zkcqlst.cloudfront.net\/content_data\/content_images\/1612329302881945835.png\"
\/>","R","None of
these"]}]}}},{"id":"911280","section_id":"45867","sequence_id":"65","sequence_oid":null,"render_gid":"","num_q
ues":"1","header":null,"content":null,"exam_id":"80","topic_id":"0","is_linked":null,"autoHeader":"","qdata":{"65"
:{"subquestion":[{"id":"993356","group_id":"911280","sequence_id":"65","sequence_oid":null,"sequence_sid":"0"
,

"render_qid":"65","render_sqid":"","header":null,"type":"objective","category":"none","is_mco":"no","level":"low
","time":"120","marks":"4","neg_marks":"1","meta_id":"3420","dirty":null,"comment":null,"history":"[<b>Feb 05
2021 12:03:32][admin<\/b>][I][<b>admin<\/b>]","answer":"1","prefix":null,"content":"The range of the function
f(x) =&nbsp;<img alt=\"frac{7}{{100 - 50cos x}}\"
src=\"https:\/\/d2lbh14zkcqlst.cloudfront.net\/content_data\/content_images\/1612348663829769702.png\"
\/>&nbsp; is :-","suffix":null,"testcases":[],"solution":null,"insert_time":"2021-02-05
12:03:32","login_id":"admin","topic_id":0,"hint":null,"qmock_list":null,"unattempt":"2","unattempt_avg_time":"1
7.67","unevaluate":"0","unevaluate_avg_time":"45","correct":"0","correct_avg_time":"45","incorrect":"0","incorr
ect_avg_time":"45","sp_answer":"","mco_marks":"0","exam_ques":"no","exam_info":null,"sub_type":null,"is_bon
us":"0","is_extra_question":"no","answer_type":null,"integer_length":null,"error_status":"0","source_q_id":"1467
676","topic":"miscellaneous","subject":"mathematics","options":["<img alt=\"left[ {frac{7}{{150}},frac{7}{{50}}}
right]\"
src=\"https:\/\/d2lbh14zkcqlst.cloudfront.net\/content_data\/content_images\/1612329978581465683.png\"
\/>","<img alt=\"left( { - infty ,frac{7}{{150}}} right] cup left[ {frac{7}{{50}},infty } right)\"
src=\"https:\/\/d2lbh14zkcqlst.cloudfront.net\/content_data\/content_images\/1612329984415974002.png\"
\/>","<img alt=\"left[ {frac{{ - 7}}{{50}},frac{{ - 7}}{{150}}} right]\"
src=\"https:\/\/d2lbh14zkcqlst.cloudfront.net\/content_data\/content_images\/16123299901443817063.png\"
\/>","<img alt=\"left( { - infty ,,,frac{{ - 7}}{{50}}} right] cup left[ {frac{{ - 7}}{{150}},infty } right)\"
src=\"https:\/\/d2lbh14zkcqlst.cloudfront.net\/content_data\/content_images\/1612329996402108292.png\"
\/>"]}]}}},{"id":"911281","section_id":"45867","sequence_id":"66","sequence_oid":null,"render_gid":"","num_que
s":"1","header":null,"content":null,"exam_id":"80","topic_id":"0","is_linked":null,"autoHeader":"","qdata":{"66":{"
subquestion":[{"id":"993357","group_id":"911281","sequence_id":"66","sequence_oid":null,"sequence_sid":"0"

,"render_qid":"66","render_sqid":"","header":null,"type":"objective","category":"none","is_mco":"no","level":"lo
w","time":"120","marks":"4","neg_marks":"1","meta_id":"3420","dirty":null,"comment":null,"history":"[<b>Feb
05 2021 12:03:32][admin<\/b>][I][<b>admin<\/b>]","answer":"3","prefix":null,"content":"Which of the following
relation is a function ?","suffix":null,"testcases":[],"solution":null,"insert_time":"2021-02-05
12:03:32","login_id":"admin","topic_id":0,"hint":null,"qmock_list":null,"unattempt":"2","unattempt_avg_time":"2
0","unevaluate":"0","unevaluate_avg_time":"45","correct":"0","correct_avg_time":"45","incorrect":"0","incorrect
_avg_time":"45","sp_answer":"","mco_marks":"0","exam_ques":"no","exam_info":null,"sub_type":null,"is_bonus"
:"0","is_extra_question":"no","answer_type":null,"integer_length":null,"error_status":"0","source_q_id":"146768
2","topic":"miscellaneous","subject":"mathematics","options":["{(1,4), (2,6), (1,5), (3,9)} &nbsp;","{(3,3), (2,1),
(1,2), (2,3)}","{(1,2), (2,2,), (3,2), (4,2)}","{(3,1), (3,2), (3,3),
(3,4)}"]}]}}},{"id":"911282","section_id":"45867","sequence_id":"67","sequence_oid":null,"render_gid":"","num_q
ues":"1","header":null,"content":null,"exam_id":"80","topic_id":"0","is_linked":null,"autoHeader":"","qdata":{"67"
:{"subquestion":[{"id":"993358","group_id":"911282","sequence_id":"67","sequence_oid":null,"sequence_sid":"0"

,"render_qid":"67","render_sqid":"","header":null,"type":"objective","category":"none","is_mco":"no","level":"hig
h","time":"120","marks":"4","neg_marks":"1","meta_id":"3420","dirty":null,"comment":null,"history":"[<b>Feb 05
2021 12:03:33][admin<\/b>][I][<b>admin<\/b>]","answer":"1","prefix":null,"content":"<div style=\"text-
align:justify\">If one n-sided regular polygon is inscribed in a circle and in another regular polygon same circle is
inscribed, then ratio of length of their respective sides is -
&nbsp;<\/div>\n","suffix":null,"testcases":[],"solution":null,"insert_time":"2021-02-05
12:03:33","login_id":"admin","topic_id":0,"hint":null,"qmock_list":null,"unattempt":"2","unattempt_avg_time":"1
7","unevaluate":"0","unevaluate_avg_time":"45","correct":"0","correct_avg_time":"45","incorrect":"0","incorrect
_avg_time":"45","sp_answer":"","mco_marks":"0","exam_ques":"no","exam_info":null,"sub_type":null,"is_bonus"
:"0","is_extra_question":"no","answer_type":null,"integer_length":null,"error_status":"0","source_q_id":"146769
4","topic":"miscellaneous","subject":"mathematics","options":["cos<img alt=\"frac{pi }{n}\"
src=\"https:\/\/d2lbh14zkcqlst.cloudfront.net\/content_data\/content_images\/1612330139934757075.png\"
\/>","sin<img alt=\"frac{pi }{n}\"
src=\"https:\/\/d2lbh14zkcqlst.cloudfront.net\/content_data\/content_images\/161233015014574821.png\"
\/>","sec<img alt=\"frac{pi }{n}\"
src=\"https:\/\/d2lbh14zkcqlst.cloudfront.net\/content_data\/content_images\/1612330158649642921.png\"
\/>","tan<img alt=\"frac{pi }{n}\"
src=\"https:\/\/d2lbh14zkcqlst.cloudfront.net\/content_data\/content_images\/1612330166840956078.png\"
\/>"]}]}}},{"id":"911283","section_id":"45867","sequence_id":"68","sequence_oid":null,"render_gid":"","num_que
s":"1","header":null,"content":null,"exam_id":"80","topic_id":"0","is_linked":null,"autoHeader":"","qdata":{"68":{"
subquestion":[{"id":"993359","group_id":"911283","sequence_id":"68","sequence_oid":null,"sequence_sid":"0"
,"render_qid":"68","render_sqid":"","header":null,"type":"objective","category":"none","is_mco":"no","level":"me
dium","time":"120","marks":"4","neg_marks":"1","meta_id":"3420","dirty":null,"comment":null,"history":"[<b>Fe
b 05 2021 12:03:34][admin<\/b>][I][<b>admin<\/b>]","answer":"4","prefix":null,"content":"A function whose
graph is symmetrical about the y&ndash;axis (on their respective domain) is given
by","suffix":null,"testcases":[],"solution":null,"insert_time":"2021-02-05
12:03:34","login_id":"admin","topic_id":0,"hint":null,"qmock_list":null,"unattempt":"2","unattempt_avg_time":"1
7.33","unevaluate":"0","unevaluate_avg_time":"45","correct":"0","correct_avg_time":"45","incorrect":"0","incorr
ect_avg_time":"45","sp_answer":"","mco_marks":"0","exam_ques":"no","exam_info":null,"sub_type":null,"is_bon
us":"0","is_extra_question":"no","answer_type":null,"integer_length":null,"error_status":"0","source_q_id":"1467
702","topic":"miscellaneous","subject":"mathematics","options":["f (x) = sin [log (x +<img alt=\"sqrt {{x^2} + 1} \"
src=\"https:\/\/d2lbh14zkcqlst.cloudfront.net\/content_data\/content_images\/161233021954361278.png\" \/>)]
&nbsp;","<img alt=\"f(x) = frac{{{{sec }^4}x, + cos e{c^4}x}}{{{x^3} + {x^4},cot x}}\"
src=\"https:\/\/d2lbh14zkcqlst.cloudfront.net\/content_data\/content_images\/16123302321403440334.png\"
\/>","f (x) = x<sup>3<\/sup> + sinx + |x|","None of
these&nbsp;"]}]}}},{"id":"911284","section_id":"45867","sequence_id":"69","sequence_oid":null,"render_gid":"","
num_ques":"1","header":null,"content":null,"exam_id":"80","topic_id":"0","is_linked":null,"autoHeader":"","qdat
a":{"69":{"subquestion":[{"id":"993360","group_id":"911284","sequence_id":"69","sequence_oid":null,"sequence
_sid":"0

","render_qid":"69","render_sqid":"","header":null,"type":"objective","category":"none","is_mco":"no","level":"lo
w","time":"120","marks":"4","neg_marks":"1","meta_id":"3420","dirty":null,"comment":null,"history":"[<b>Feb
05 2021 12:03:36][admin<\/b>][I][<b>admin<\/b>]","answer":"4","prefix":null,"content":"Which of the following
functions are identical function :","suffix":null,"testcases":[],"solution":null,"insert_time":"2021-02-05
12:03:36","login_id":"admin","topic_id":0,"hint":null,"qmock_list":null,"unattempt":"2","unattempt_avg_time":"1
6.33","unevaluate":"0","unevaluate_avg_time":"45","correct":"0","correct_avg_time":"45","incorrect":"0","incorr
ect_avg_time":"45","sp_answer":"","mco_marks":"0","exam_ques":"no","exam_info":null,"sub_type":null,"is_bon
us":"0","is_extra_question":"no","answer_type":null,"integer_length":null,"error_status":"0","source_q_id":"1467
720","topic":"miscellaneous","subject":"mathematics","options":["&fnof; (x) =&nbsp;<img alt=\"sqrt {{x^2}} \"
src=\"https:\/\/d2lbh14zkcqlst.cloudfront.net\/content_data\/content_images\/16123303032123256668.png\"
\/>; g(x) =&nbsp;<img alt=\"{(sqrt x )^2}\"
src=\"https:\/\/d2lbh14zkcqlst.cloudfront.net\/content_data\/content_images\/1612330315238501468.png\"
\/>","&fnof; (x) = sec<sup>2<\/sup>x &ndash; tan<sup>2<\/sup>x; &nbsp;g(x) = 1","&fnof; (x) =&nbsp;<img
alt=\"sqrt {{x^2} - 1} \"
src=\"https:\/\/d2lbh14zkcqlst.cloudfront.net\/content_data\/content_images\/1612330343876458789.png\"
\/>&nbsp;; g(x) =&nbsp;<img alt=\"sqrt {x - 1} .sqrt {x + 1} \"
src=\"https:\/\/d2lbh14zkcqlst.cloudfront.net\/content_data\/content_images\/161233035519517938.png\"
\/>","<img alt=\"f(x) = {text{cosecx}},,{text{;}},,,g(x) = frac{1}{{sin x}}\"
src=\"https:\/\/d2lbh14zkcqlst.cloudfront.net\/content_data\/content_images\/16123303621213558668.png\"
\/>"]}]}}},{"id":"911285","section_id":"45867","sequence_id":"70","sequence_oid":null,"render_gid":"","num_que
s":"1","header":null,"content":null,"exam_id":"80","topic_id":"0","is_linked":null,"autoHeader":"","qdata":{"70":{"
subquestion":[{"id":"993361","group_id":"911285","sequence_id":"70","sequence_oid":null,"sequence_sid":"0
","render_qid":"70","render_sqid":"","header":null,"type":"objective","category":"none","is_mco":"no","level":"lo
w","time":"120","marks":"4","neg_marks":"1","meta_id":"3420","dirty":null,"comment":null,"history":"[<b>Feb
05 2021 12:03:36][admin<\/b>][I][<b>admin<\/b>]","answer":"1","prefix":null,"content":"The function &nbsp;f :
R&ndash;{0} &rarr; R defined by&nbsp;<img alt=\"f(x) = frac{x}{{1 - {2^x}}} - frac{x}{2}\"
src=\"https:\/\/d2lbh14zkcqlst.cloudfront.net\/content_data\/content_images\/16123304032073279275.png\"
\/>&nbsp;&nbsp;is&nbsp;","suffix":null,"testcases":[],"solution":null,"insert_time":"2021-02-05
12:03:36","login_id":"admin","topic_id":0,"hint":null,"qmock_list":null,"unattempt":"2","unattempt_avg_time":"1
7","unevaluate":"0","unevaluate_avg_time":"45","correct":"0","correct_avg_time":"45","incorrect":"0","incorrect
_avg_time":"45","sp_answer":"","mco_marks":"0","exam_ques":"no","exam_info":null,"sub_type":null,"is_bonus"
:"0","is_extra_question":"no","answer_type":null,"integer_length":null,"error_status":"0","source_q_id":"146772
9","topic":"miscellaneous","subject":"mathematics","options":["an even function","an odd function","neither even
nor odd function","None of
these&nbsp;"]}]}}},{"id":"911286","section_id":"45867","sequence_id":"71","sequence_oid":null,"render_gid":"","
num_ques":"1","header":null,"content":null,"exam_id":"80","topic_id":"0","is_linked":null,"autoHeader":"","qdat
a":{"71":{"subquestion":[{"id":"993362","group_id":"911286","sequence_id":"71","sequence_oid":null,"sequence
_sid":"0"

,"render_qid":"71","render_sqid":"","header":null,"type":"objective","category":"none","is_mco":"no","level":"lo
w","time":"120","marks":"4","neg_marks":"1","meta_id":"3420","dirty":null,"comment":null,"history":"[<b>Feb
05 2021 12:03:36][admin<\/b>][I][<b>admin<\/b>]","answer":"4","prefix":null,"content":"Let &nbsp;f (x) =
(x<sup>12<\/sup> - x<sup>9<\/sup> + x<sup>4<\/sup> - x + 1)<sup>-1\/2<\/sup> . The domain of the function is
:","suffix":null,"testcases":[],"solution":null,"insert_time":"2021-02-05
12:03:36","login_id":"admin","topic_id":0,"hint":null,"qmock_list":null,"unattempt":"2","unattempt_avg_time":"1
7.67","unevaluate":"0","unevaluate_avg_time":"45","correct":"0","correct_avg_time":"45","incorrect":"0","incorr
ect_avg_time":"45","sp_answer":"","mco_marks":"0","exam_ques":"no","exam_info":null,"sub_type":null,"is_bon
us":"0","is_extra_question":"no","answer_type":null,"integer_length":null,"error_status":"0","source_q_id":"1467
741","topic":"miscellaneous","subject":"mathematics","options":["(1 , &infin;)","(- &infin; , - 1)","(- 1 , 1)","(-
&infin; ,
&infin;)"]}]}}},{"id":"911287","section_id":"45867","sequence_id":"72","sequence_oid":null,"render_gid":"","num_
ques":"1","header":null,"content":null,"exam_id":"80","topic_id":"0","is_linked":null,"autoHeader":"","qdata":{"7
2":{"subquestion":[{"id":"993363","group_id":"911287","sequence_id":"72","sequence_oid":null,"sequence_sid":"
0"

,"render_qid":"72","render_sqid":"","header":null,"type":"objective","category":"none","is_mco":"no","level":"me
dium","time":"120","marks":"4","neg_marks":"1","meta_id":"3420","dirty":null,"comment":null,"history":"[<b>Fe
b 05 2021 12:03:37][admin<\/b>][I][<b>admin<\/b>]","answer":"3","prefix":null,"content":"Let a and b are real
numbers and let&nbsp;<br \/>\n<img alt=\"f(x) = asin x + broot 3 of x \u00a0+ 4\"
src=\"https:\/\/d2lbh14zkcqlst.cloudfront.net\/content_data\/content_images\/16123305552053041040.png\"
\/><br \/>\nif f(log<sub>10<\/sub>log<sub>3<\/sub>10) = 5 then f(log<sub>10<\/sub>log<sub>10<\/sub>3)
=","suffix":null,"testcases":[],"solution":null,"insert_time":"2021-02-05
12:03:37","login_id":"admin","topic_id":0,"hint":null,"qmock_list":null,"unattempt":"2","unattempt_avg_time":"1
9","unevaluate":"0","unevaluate_avg_time":"45","correct":"0","correct_avg_time":"45","incorrect":"0","incorrect
_avg_time":"45","sp_answer":"","mco_marks":"0","exam_ques":"no","exam_info":null,"sub_type":null,"is_bonus"
:"0","is_extra_question":"no","answer_type":null,"integer_length":null,"error_status":"0","source_q_id":"146774
7","topic":"miscellaneous","subject":"mathematics","options":["&ndash;5","&ndash;3","3","Depends an a,
b"]}]}}},{"id":"911288","section_id":"45867","sequence_id":"73","sequence_oid":null,"render_gid":"","num_ques"
:"1","header":null,"content":null,"exam_id":"80","topic_id":"0","is_linked":null,"autoHeader":"","qdata":{"73":{"s
ubquestion":[{"id":"993364","group_id":"911288","sequence_id":"73","sequence_oid":null,"sequence_sid":"0

","render_qid":"73","render_sqid":"","header":null,"type":"objective","category":"none","is_mco":"no","level":"lo
w","time":"120","marks":"4","neg_marks":"1","meta_id":"3420","dirty":null,"comment":null,"history":"[<b>Feb
05 2021 12:03:37][admin<\/b>][I][<b>admin<\/b>]","answer":"3","prefix":null,"content":"Let f(x) be such that f(x +
2) = f(x) and &nbsp;f(&ndash;x) = f(x) &forall; x &isin; R. On the interval [2, 3], f(x) = x. Then which of the following
is correct?","suffix":null,"testcases":[],"solution":null,"insert_time":"2021-02-05
12:03:37","login_id":"admin","topic_id":0,"hint":null,"qmock_list":null,"unattempt":"2","unattempt_avg_time":"1
9","unevaluate":"0","unevaluate_avg_time":"45","correct":"0","correct_avg_time":"45","incorrect":"0","incorrect
_avg_time":"45","sp_answer":"","mco_marks":"0","exam_ques":"no","exam_info":null,"sub_type":null,"is_bonus"
:"0","is_extra_question":"no","answer_type":null,"integer_length":null,"error_status":"0","source_q_id":"146776
8","topic":"miscellaneous","subject":"mathematics","options":["f(x) = x + 4, &forall; x &isin; [&ndash;2, 0]","f(x) = 2
&ndash; x, &forall; x &isin; [&ndash;2, 0]","f(x) = 3 &ndash; |x + 1|, &forall; x &isin; [&ndash;2, 0]","f(x) = 2 + |x +
1|, &forall; x &isin; [&ndash;2,
0]"]}]}}},{"id":"911289","section_id":"45867","sequence_id":"74","sequence_oid":null,"render_gid":"","num_ques
":"1","header":null,"content":null,"exam_id":"80","topic_id":"0","is_linked":null,"autoHeader":"","qdata":{"74":{"s
ubquestion":[{"id":"993365","group_id":"911289","sequence_id":"74","sequence_oid":null,"sequence_sid":"0",

"render_qid":"74","render_sqid":"","header":null,"type":"objective","category":"none","is_mco":"no","level":"low
","time":"120","marks":"4","neg_marks":"1","meta_id":"3420","dirty":null,"comment":null,"history":"[<b>Feb 05
2021 12:03:38][admin<\/b>][I][<b>admin<\/b>]","answer":"2","prefix":null,"content":"Let f(x) be such
that&nbsp;<img alt=\"fleft( {frac{{1 - x}}{{1 + x}}} right) = x\"
src=\"https:\/\/d2lbh14zkcqlst.cloudfront.net\/content_data\/content_images\/16123307531991519140.png\"
\/>&nbsp;, &forall; x &isin; R &ndash; {&ndash;1}. Then which of the following is
correct&nbsp;","suffix":null,"testcases":[],"solution":null,"insert_time":"2021-02-05
12:03:38","login_id":"admin","topic_id":0,"hint":null,"qmock_list":null,"unattempt":"2","unattempt_avg_time":"1
9","unevaluate":"0","unevaluate_avg_time":"45","correct":"0","correct_avg_time":"45","incorrect":"0","incorrect
_avg_time":"45","sp_answer":"","mco_marks":"0","exam_ques":"no","exam_info":null,"sub_type":null,"is_bonus"
:"0","is_extra_question":"no","answer_type":null,"integer_length":null,"error_status":"0","source_q_id":"146779
0","topic":"miscellaneous","subject":"mathematics","options":["<img alt=\"fleft( {frac{1}{x}} right) = f(x),forall x in
R - { 0} \"
src=\"https:\/\/d2lbh14zkcqlst.cloudfront.net\/content_data\/content_images\/1612330844211209826.png\"
\/>","<img alt=\"fleft( {frac{1}{x}} right) = \u00a0- f(x),forall x in R - { 0} \"
src=\"https:\/\/d2lbh14zkcqlst.cloudfront.net\/content_data\/content_images\/16123308511387037019.png\"
\/>","<img alt=\"fleft( {frac{1}{x}} right) = 2f(x),forall x in R - { 0} \"
src=\"https:\/\/d2lbh14zkcqlst.cloudfront.net\/content_data\/content_images\/1612330858732050891.png\"
\/>","<img alt=\"fleft( {frac{1}{x}} right) = \u00a0- 2f(x),forall x in R - { 0} \"
src=\"https:\/\/d2lbh14zkcqlst.cloudfront.net\/content_data\/content_images\/1612330865638182289.png\"
\/>"]}]}}},{"id":"911290","section_id":"45867","sequence_id":"75","sequence_oid":null,"render_gid":"","num_que
s":"1","header":null,"content":null,"exam_id":"80","topic_id":"0","is_linked":null,"autoHeader":"","qdata":{"75":{"
subquestion":[{"id":"993366","group_id":"911290","sequence_id":"75","sequence_oid":null,"sequence_sid":"0"

,"render_qid":"75","render_sqid":"","header":null,"type":"objective","category":"none","is_mco":"no","level":"lo
w","time":"120","marks":"4","neg_marks":"1","meta_id":"3420","dirty":null,"comment":null,"history":"[<b>Feb
05 2021 12:03:38][admin<\/b>][I][<b>admin<\/b>]","answer":"1","prefix":null,"content":"A function f &nbsp;has
domain [&ndash; 1, 2] and range [0, 1]. The domain and range respectively of the function g defined by g (x) = 2
&ndash; f (x + 3) is","suffix":null,"testcases":[],"solution":null,"insert_time":"2021-02-05
12:03:38","login_id":"admin","topic_id":0,"hint":null,"qmock_list":null,"unattempt":"2","unattempt_avg_time":"1
8.33","unevaluate":"0","unevaluate_avg_time":"45","correct":"0","correct_avg_time":"45","incorrect":"0","incorr
ect_avg_time":"45","sp_answer":"","mco_marks":"0","exam_ques":"no","exam_info":null,"sub_type":null,"is_bon
us":"0","is_extra_question":"no","answer_type":null,"integer_length":null,"error_status":"0","source_q_id":"1467
804","topic":"miscellaneous","subject":"mathematics","options":["[&ndash;4, &ndash;1] ; [1, 2]","[&ndash;2, 1];
[0, 1]","[0, 2] ; [&ndash;1, 0]","None of
these"]}]}}},{"id":"911291","section_id":"45867","sequence_id":"76","sequence_oid":null,"render_gid":"","num_q
ues":"1","header":null,"content":null,"exam_id":"80","topic_id":"0","is_linked":null,"autoHeader":"","qdata":{"76"
:{"subquestion":[{"id":"993367","group_id":"911291","sequence_id":"76","sequence_oid":null,"sequence_sid":"0"
,

"render_qid":"76","render_sqid":"","header":null,"type":"objective","category":"none","is_mco":"no","level":"me
dium","time":"120","marks":"4","neg_marks":"1","meta_id":"3420","dirty":null,"comment":null,"history":"[<b>Fe
b 05 2021 12:03:39][admin<\/b>][I][<b>admin<\/b>]","answer":"3","prefix":null,"content":"Number of integral
values of x satisfying&nbsp;<img alt=\"frac{{left( {sqrt {x + 1} } right)left( {{x^2} - 4x + 3} right)}}{{left( {x + 3} right)}}
leqslant 0\"
src=\"https:\/\/d2lbh14zkcqlst.cloudfront.net\/content_data\/content_images\/16123310281242048904.png\"
\/>&nbsp;is\/are","suffix":null,"testcases":[],"solution":null,"insert_time":"2021-02-05
12:03:39","login_id":"admin","topic_id":0,"hint":null,"qmock_list":null,"unattempt":"2","unattempt_avg_time":"1
6.67","unevaluate":"0","unevaluate_avg_time":"45","correct":"0","correct_avg_time":"45","incorrect":"0","incorr
ect_avg_time":"45","sp_answer":"","mco_marks":"0","exam_ques":"no","exam_info":null,"sub_type":null,"is_bon
us":"0","is_extra_question":"no","answer_type":null,"integer_length":null,"error_status":"0","source_q_id":"1467
811","topic":"miscellaneous","subject":"mathematics","options":["2","3","4","infinite"]}]}}},{"id":"911292","sectio
n_id":"45867","sequence_id":"77","sequence_oid":null,"render_gid":"","num_ques":"1","header":null,"content":n
ull,"exam_id":"80","topic_id":"0","is_linked":null,"autoHeader":"","qdata":{"77":{"subquestion":[{"id":"993368","g
roup_id":"911292","sequence_id":"77","sequence_oid":null,"sequence_sid":"0

","render_qid":"77","render_sqid":"","header":null,"type":"objective","category":"none","is_mco":"no","level":"lo
w","time":"120","marks":"4","neg_marks":"1","meta_id":"3420","dirty":null,"comment":null,"history":"[<b>Feb
05 2021 12:03:39][admin<\/b>][I][<b>admin<\/b>]","answer":"3","prefix":null,"content":"In a triangle ABC, (a + b
+ c) (b + c &ndash; a) = &lambda;bc if -","suffix":null,"testcases":[],"solution":null,"insert_time":"2021-02-05
12:03:39","login_id":"admin","topic_id":0,"hint":null,"qmock_list":null,"unattempt":"2","unattempt_avg_time":"1
7.33","unevaluate":"0","unevaluate_avg_time":"45","correct":"0","correct_avg_time":"45","incorrect":"0","incorr
ect_avg_time":"45","sp_answer":"","mco_marks":"0","exam_ques":"no","exam_info":null,"sub_type":null,"is_bon
us":"0","is_extra_question":"no","answer_type":null,"integer_length":null,"error_status":"0","source_q_id":"1467
814","topic":"miscellaneous","subject":"mathematics","options":["&lambda; &lt; 0","&lambda; &gt; 0","0 &lt;
&lambda; &lt; 4","&lambda; &gt;
4"]}]}}},{"id":"911293","section_id":"45867","sequence_id":"78","sequence_oid":null,"render_gid":"","num_ques"
:"1","header":null,"content":null,"exam_id":"80","topic_id":"0","is_linked":null,"autoHeader":"","qdata":{"78":{"s
ubquestion":[{"id":"993369","group_id":"911293","sequence_id":"78","sequence_oid":null,"sequence_sid":"0"

,"render_qid":"78","render_sqid":"","header":null,"type":"objective","category":"none","is_mco":"no","level":"lo
w","time":"120","marks":"4","neg_marks":"1","meta_id":"3420","dirty":null,"comment":null,"history":"[<b>Feb
05 2021 12:03:40][admin<\/b>][I][<b>admin<\/b>]","answer":"1","prefix":null,"content":"The sides of a triangle
ABC are&nbsp;<img alt=\"x,y,sqrt {{x^2} + {y^2} + xy} \"
src=\"https:\/\/d2lbh14zkcqlst.cloudfront.net\/content_data\/content_images\/16123311351163945507.png\"
\/>&nbsp;respectively. The size of the greatest angle in radians
is&nbsp;","suffix":null,"testcases":[],"solution":null,"insert_time":"2021-02-05
12:03:40","login_id":"admin","topic_id":0,"hint":null,"qmock_list":null,"unattempt":"2","unattempt_avg_time":"1
7.33","unevaluate":"0","unevaluate_avg_time":"45","correct":"0","correct_avg_time":"45","incorrect":"0","incorr
ect_avg_time":"45","sp_answer":"","mco_marks":"0","exam_ques":"no","exam_info":null,"sub_type":null,"is_bon
us":"0","is_extra_question":"no","answer_type":null,"integer_length":null,"error_status":"0","source_q_id":"1467
820","topic":"miscellaneous","subject":"mathematics","options":["<img alt=\"frac{{2pi }}{3}\"
src=\"https:\/\/d2lbh14zkcqlst.cloudfront.net\/content_data\/content_images\/1612331144268445958.png\"
\/>","<img alt=\"frac{pi }{3}\"
src=\"https:\/\/d2lbh14zkcqlst.cloudfront.net\/content_data\/content_images\/1612331149668358356.png\"
\/>","<img alt=\"frac{pi }{2}\"
src=\"https:\/\/d2lbh14zkcqlst.cloudfront.net\/content_data\/content_images\/16123311541216965988.png\"
\/>","None of
these"]}]}}},{"id":"911294","section_id":"45867","sequence_id":"79","sequence_oid":null,"render_gid":"","num_q
ues":"1","header":null,"content":null,"exam_id":"80","topic_id":"0","is_linked":null,"autoHeader":"","qdata":{"79"
:{"subquestion":[{"id":"993370","group_id":"911294","sequence_id":"79","sequence_oid":null,"sequence_sid":"0"
,

"render_qid":"79","render_sqid":"","header":null,"type":"objective","category":"none","is_mco":"no","level":"me
dium","time":"120","marks":"4","neg_marks":"1","meta_id":"3420","dirty":null,"comment":null,"history":"[<b>Fe
b 05 2021 12:03:41][admin<\/b>][I][<b>admin<\/b>]","answer":"2","prefix":null,"content":"If the function f : [1,
&infin;) &nbsp;[1, &infin;) is defined by f(x) = 2<sup>x(x &ndash; 1)<\/sup>, then f <sup>&ndash;1<\/sup>(x) is -
","suffix":null,"testcases":[],"solution":null,"insert_time":"2021-02-05
12:03:41","login_id":"admin","topic_id":0,"hint":null,"qmock_list":null,"unattempt":"2","unattempt_avg_time":"1
8.33","unevaluate":"0","unevaluate_avg_time":"45","correct":"0","correct_avg_time":"45","incorrect":"0","incorr
ect_avg_time":"45","sp_answer":"","mco_marks":"0","exam_ques":"no","exam_info":null,"sub_type":null,"is_bon
us":"0","is_extra_question":"no","answer_type":null,"integer_length":null,"error_status":"0","source_q_id":"1467
830","topic":"miscellaneous","subject":"mathematics","options":["<img alt=\"{left( {frac{1}{2}} right)^{x(x - 1)}}\"
src=\"https:\/\/d2lbh14zkcqlst.cloudfront.net\/content_data\/content_images\/1612331226997166712.png\"
\/>","<img alt=\"frac{1}{2}left( {1 + sqrt {1 + 4{{log }_2}x} } right)\"
src=\"https:\/\/d2lbh14zkcqlst.cloudfront.net\/content_data\/content_images\/16123312322090017455.png\"
\/>","<img alt=\"frac{1}{2}left( {1 - sqrt {1 + 4{{log }_2}x} } right)\"
src=\"https:\/\/d2lbh14zkcqlst.cloudfront.net\/content_data\/content_images\/161233123722766403.png\"
\/>","Not
defined"]}]}}},{"id":"911295","section_id":"45867","sequence_id":"80","sequence_oid":null,"render_gid":"","num_
ques":"1","header":null,"content":null,"exam_id":"80","topic_id":"0","is_linked":null,"autoHeader":"","qdata":{"8
0":{"subquestion":[{"id":"993371","group_id":"911295","sequence_id":"80","sequence_oid":null,"sequence_sid":"
0"

,"render_qid":"80","render_sqid":"","header":null,"type":"objective","category":"none","is_mco":"no","level":"lo
w","time":"120","marks":"4","neg_marks":"1","meta_id":"3420","dirty":null,"comment":null,"history":"[<b>Feb
05 2021 12:03:41][admin<\/b>][I][<b>admin<\/b>]","answer":"2","prefix":null,"content":"If in a triangle PQR, sin
P, sin Q, sin R are in A.P., then -&nbsp;","suffix":null,"testcases":[],"solution":null,"insert_time":"2021-02-05
12:03:41","login_id":"admin","topic_id":0,"hint":null,"qmock_list":null,"unattempt":"2","unattempt_avg_time":"1
8.33","unevaluate":"0","unevaluate_avg_time":"45","correct":"0","correct_avg_time":"45","incorrect":"0","incorr
ect_avg_time":"45","sp_answer":"","mco_marks":"0","exam_ques":"no","exam_info":null,"sub_type":null,"is_bon
us":"0","is_extra_question":"no","answer_type":null,"integer_length":n ions":["the altitudes are in A.P.","the
altitudes are in H.P.","the medians are in G.P.&nbsp;","the medians are
ull,"error_status":"0","source_q_id":"1467842","topic":"miscellaneous","subject":"mathematics","opt in
A.P."]}]}}}]},{"id":"45868","paper_id":"12803","section":"Mathematics","subsection":"SEC-
B","sequence_id":"81","sequence_sid":"0","header":"<div>\r\n<ul>\r\n\t<li>This section has <b>TEN<\/b>
questions.<\/li>\r\n\t<li>Candidates have to <b>attempt any 5 questions out of 10<\/b>. If more than 5 questions
are attempted, then only first 5 attempted questions will be evaluated.<\/li>\r\n\t<li>The answer to each question
is a <b>NUMERICAL VALUE<\/b>.<\/li>\r\n\t<li>For each question, enter the correct numerical value (in decimal
notation, truncated\/rounded-off to the <b>second decimal place <\/b>e.g. 6.25, 7.00, &ndash;0.33, &ndash;.30,
30.27, &ndash;127.30, if answer is 11.36777..... then both 11.36 and 11.37 will be correct).<\/li>\r\n\t<li>For each
question, type the correct integer in the space provided below the question using provided number
keys.<\/li>\r\n\t<li><b>Marking scheme:<\/b>\r\n\t<ul>\r\n\t\t<li>Full Marks : +4 If the correct integer is typed in
the provided space.<\/li>\r\n\t\t<li>Zero Marks : 0 In all other
cases.<\/li>\r\n\t<\/ul>\r\n\t<\/li>\r\n<\/ul>\r\n<\/div>\r\n","num_grps":"10","marks":"20","neg_marks":"0","ti
me_duration":"20","type":"paper","ans_limit":"5","gdata":[{"id":"911296","section_id":"45868","sequence_id":"8
1","sequence_oid":null,"render_gid":"","num_ques":"1","header":null,"content":null,"exam_id":"80","topic_id":"0
","is_linked":null,"autoHeader":"","qdata":{"81":{"subquestion":[{"id":"993372","group_id":"911296","sequence_i
d":"81","sequence_oid":null,"sequence_sid":"0"

,"render_qid":"81","render_sqid":"","header":null,"type":"subjective","category":"none","is_mco":"no","level":"lo
w","time":"120","marks":"4","neg_marks":"0","meta_id":"3420","dirty":null,"comment":null,"history":"[<b>Feb
05 2021 12:03:41][admin<\/b>][I][<b>admin<\/b>]","answer":"7.00","prefix":null,"content":"In a triangle ABC, If a
= 9, b = 8 and c = x satisfies 3 cos C = 2, then x =","suffix":null,"testcases":[],"solution":null,"insert_time":"2021-02-
05
12:03:41","login_id":"admin","topic_id":0,"hint":null,"qmock_list":null,"unattempt":"0","unattempt_avg_time":"4
5","unevaluate":"0","unevaluate_avg_time":"45","correct":"0","correct_avg_time":"45","incorrect":"2","incorrect
_avg_time":"22.33","sp_answer":"","mco_marks":"0","exam_ques":"no","exam_info":null,"sub_type":"numerical"
,"is_bonus":"0","is_extra_question":"no","answer_type":null,"integer_length":null,"error_status":"0","source_q_i
d":"1467846","topic":"miscellaneous","subject":"mathematics"}]}}},{"id":"911297","section_id":"45868","sequenc
e_id":"82","sequence_oid":null,"render_gid":"","num_ques":"1","header":null,"content":null,"exam_id":"80","topi
c_id":"0","is_linked":null,"autoHeader":"","qdata":{"82":{"subquestion":[{"id":"993373","group_id":"911297","seq
uence_id":"82","sequence_oid":null,"sequence_sid":"0"

,"render_qid":"82","render_sqid":"","header":null,"type":"subjective","category":"none","is_mco":"no","level":"lo
w","time":"120","marks":"4","neg_marks":"0","meta_id":"3420","dirty":null,"comment":null,"history":"[<b>Feb
05 2021 12:03:42][admin<\/b>][I][<b>admin<\/b>]","answer":"1.00","prefix":null,"content":"In a triangle &ang;A =
55&deg; and &ang;B = 15&ordm;, then&nbsp;<img alt=\"frac{{{c^2} - {a^2}}}{{ab}}\"
src=\"https:\/\/d2lbh14zkcqlst.cloudfront.net\/content_data\/content_images\/16123314091722894203.png\"
\/>&nbsp;&nbsp;is equal to -","suffix":null,"testcases":[],"solution":null,"insert_time":"2021-02-05
12:03:42","login_id":"admin","topic_id":0,"hint":null,"qmock_list":null,"unattempt":"2","unattempt_avg_time":"1
5.67","unevaluate":"0","unevaluate_avg_time":"45","correct":"0","correct_avg_time":"45","incorrect":"0","incorr
ect_avg_time":"45","sp_answer":"","mco_marks":"0","exam_ques":"no","exam_info":null,"sub_type":"numerical"
,"is_bonus":"0","is_extra_question":"no","answer_type":null,"integer_length":null,"error_status":"0","source_q_i
d":"1467851","topic":"miscellaneous","subject":"mathematics"}]}}},{"id":"911298","section_id":"45868","sequenc
e_id":"83","sequence_oid":null,"render_gid":"","num_ques":"1","header":null,"content":null,"exam_id":"80","topi
c_id":"0","is_linked":null,"autoHeader":"","qdata":{"83":{"subquestion":[{"id":"993374","group_id":"911298","seq
uence_id":"83","sequence_oid":null,"sequence_sid":"0"

,"render_qid":"83","render_sqid":"","header":null,"type":"subjective","category":"none","is_mco":"no","level":"lo
w","time":"120","marks":"4","neg_marks":"0","meta_id":"3420","dirty":null,"comment":null,"history":"[<b>Feb
05 2021 12:03:42][admin<\/b>][I][<b>admin<\/b>]","answer":"0.00","prefix":null,"content":"In a triangle ABC
&nbsp;the value of a sin(B &ndash; C) + b sin(C &ndash; A) + c sin(A &ndash; B) is (where a,b,c are the length of
side BC, CA, AB respectively)","suffix":null,"testcases":[],"solution":null,"insert_time":"2021-02-05
12:03:42","login_id":"admin","topic_id":0,"hint":null,"qmock_list":null,"unattempt":"2","unattempt_avg_time":"1
7","unevaluate":"0","unevaluate_avg_time":"45","correct":"0","correct_avg_time":"45","incorrect":"0","incorrect
_avg_time":"45","sp_answer":"","mco_marks":"0","exam_ques":"no","exam_info":null,"sub_type":"numerical","is
_bonus":"0","is_extra_question":"no","answer_type":null,"integer_length":null,"error_status":"0","source_q_id":"
1467855","topic":"miscellaneous","subject":"mathematics"}]}}},{"id":"911299","section_id":"45868","sequence_id
":"84","sequence_oid":null,"render_gid":"","num_ques":"1","header":null,"content":null,"exam_id":"80","topic_id
":"0","is_linked":null,"autoHeader":"","qdata":{"84":{"subquestion":[{"id":"993375","group_id":"911299","sequen
ce_id":"84","sequence_oid":null,"sequence_sid":"0

","render_qid":"84","render_sqid":"","header":null,"type":"subjective","category":"none","is_mco":"no","level":"l
ow","time":"120","marks":"4","neg_marks":"0","meta_id":"3420","dirty":null,"comment":null,"history":"[<b>Feb
05 2021 12:03:42][admin<\/b>][I][<b>admin<\/b>]","answer":"1.00","prefix":null,"content":"Number of integral
values &nbsp;in range of the function f : R &rarr; R defined by&nbsp;<img alt=\"f(x) = frac{x}{{1 + {x^2}}}\"
src=\"https:\/\/d2lbh14zkcqlst.cloudfront.net\/content_data\/content_images\/161233154057361801.png\"
\/>&nbsp;&nbsp;is &nbsp;&nbsp; &nbsp;","suffix":null,"testcases":[],"solution":null,"insert_time":"2021-02-05
12:03:42","login_id":"admin","topic_id":0,"hint":null,"qmock_list":null,"unattempt":"2","unattempt_avg_time":"1
6.67","unevaluate":"0","unevaluate_avg_time":"45","correct":"0","correct_avg_time":"45","incorrect":"0","incorr
ect_avg_time":"45","sp_answer":"","mco_marks":"0","exam_ques":"no","exam_info":null,"sub_type":"numerical"
,"is_bonus":"0","is_extra_question":"no","answer_type":null,"integer_length":null,"error_status":"0","source_q_i
d":"1467864","topic":"miscellaneous","subject":"mathematics"}]}}},{"id":"911300","section_id":"45868","sequenc
e_id":"85","sequence_oid":null,"render_gid":"","num_ques":"1","header":null,"content":null,"exam_id":"80","topi
c_id":"0","is_linked":null,"autoHeader":"","qdata":{"85":{"subquestion":[{"id":"993376","group_id":"911300","seq
uence_id":"85","sequence_oid":null,"sequence_sid":"0

","render_qid":"85","render_sqid":"","header":null,"type":"subjective","category":"none","is_mco":"no","level":"
medium","time":"120","marks":"4","neg_marks":"0","meta_id":"3420","dirty":null,"comment":null,"history":"[<b>
Feb 05 2021 12:03:43][admin<\/b>][I][<b>admin<\/b>]","answer":"2.00","prefix":null,"content":"<div style=\"text-
align:justify\">If number of integral values of x satisfying&nbsp;<img alt=\"left| {{x^2} + 3x - 4} right| + left| {{x^2}
- 7x + 10} right| = 2left| {5x - 7} right|\"
src=\"https:\/\/d2lbh14zkcqlst.cloudfront.net\/content_data\/content_images\/16123315851617022837.png\"
\/>&nbsp;is k then number of prime numbers which divide k
is\/are<\/div>\n","suffix":null,"testcases":[],"solution":null,"insert_time":"2021-02-05
12:03:43","login_id":"admin","topic_id":0,"hint":null,"qmock_list":null,"unattempt":"2","unattempt_avg_time":"1
7.67","unevaluate":"0","unevaluate_avg_time":"45","correct":"0","correct_avg_time":"45","incorrect":"0","incorr
ect_avg_time":"45","sp_answer":"","mco_marks":"0","exam_ques":"no","exam_info":null,"sub_type":"numerical"
,"is_bonus":"0","is_extra_question":"no","answer_type":null,"integer_length":null,"error_status":"0","source_q_i
d":"1467869","topic":"miscellaneous","subject":"mathematics"}]}}},{"id":"911301","section_id":"45868","sequenc
e_id":"86","sequence_oid":null,"render_gid":"","num_ques":"1","header":null,"content":null,"exam_id":"80","topi
c_id":"0","is_linked":null,"autoHeader":"","qdata":{"86":{"subquestion":[{"id":"993377","group_id":"911301","seq
uence_id":"86","sequence_oid":null,"sequence_sid":"0"

,"render_qid":"86","render_sqid":"","header":null,"type":"subjective","category":"none","is_mco":"no","level":"lo
w","time":"120","marks":"4","neg_marks":"0","meta_id":"3420","dirty":null,"comment":null,"history":"[<b>Feb
05 2021 12:03:43][admin<\/b>][I][<b>admin<\/b>]","answer":"4.00","prefix":null,"content":"If fundamental
period of f(x) is T where f(x) = |sin<sup>3<\/sup> 2x| + |cos<sup>3<\/sup> 2x|. Then&nbsp;<img alt=\"frac{pi
}{T}\"
src=\"https:\/\/d2lbh14zkcqlst.cloudfront.net\/content_data\/content_images\/16123316251863142028.png\"
\/>&nbsp;&nbsp;is&nbsp;","suffix":null,"testcases":[],"solution":null,"insert_time":"2021-02-05
12:03:43","login_id":"admin","topic_id":0,"hint":null,"qmock_list":null,"unattempt":"2","unattempt_avg_time":"1
6.33","unevaluate":"0","unevaluate_avg_time":"45","correct":"0","correct_avg_time":"45","incorrect":"0","incorr
ect_avg_time":"45","sp_answer":"","mco_marks":"0","exam_ques":"no","exam_info":null,"sub_type":"numerical"
,"is_bonus":"0","is_extra_question":"no","answer_type":null,"integer_length":null,"error_status":"0","source_q_i
d":"1467882","topic":"miscellaneous","subject":"mathematics"}]}}},{"id":"911302","section_id":"45868","sequenc
e_id":"87","sequence_oid":null,"render_gid":"","num_ques":"1","header":null,"content":null,"exam_id":"80","topi
c_id":"0","is_linked":null,"autoHeader":"","qdata":{"87":{"subquestion":[{"id":"993378","group_id":"911302","seq
uence_id":"87","sequence_oid":null,"sequence_sid":"0"
,"render_qid":"87","render_sqid":"","header":null,"type":"subjective","category":"none","is_mco":"no","level":"lo
w","time":"120","marks":"4","neg_marks":"0","meta_id":"3420","dirty":null,"comment":null,"history":"[<b>Feb
05 2021 12:03:43][admin<\/b>][I][<b>admin<\/b>]","answer":"1.00","prefix":null,"content":"The fundamental
period of function f(x) = sin(x+3 &ndash;[x+3]) where [.] denotes greatest integer function
is&nbsp;","suffix":null,"testcases":[],"solution":null,"insert_time":"2021-02-05
12:03:43","login_id":"admin","topic_id":0,"hint":null,"qmock_list":null,"unattempt":"2","unattempt_avg_time":"1
8.33","unevaluate":"0","unevaluate_avg_time":"45","correct":"0","correct_avg_time":"45","incorrect":"0","incorr
ect_avg_time":"45","sp_answer":"","mco_marks":"0","exam_ques":"no","exam_info":null,"sub_type":"numerical"
,"is_bonus":"0","is_extra_question":"no","answer_type":null,"integer_length":null,"error_status":"0","source_q_i
d":"1467887","topic":"miscellaneous","subject":"mathematics"}]}}},{"id":"911303","section_id":"45868","sequenc
e_id":"88","sequence_oid":null,"render_gid":"","num_ques":"1","header":null,"content":null,"exam_id":"80","topi
c_id":"0","is_linked":null,"autoHeader":"","qdata":{"88":{"subquestion":[{"id":"993379","group_id":"911303","seq
uence_id":"88","sequence_oid":null,"sequence_sid":"0

","render_qid":"88","render_sqid":"","header":null,"type":"subjective","category":"none","is_mco":"no","level":"l
ow","time":"120","marks":"4","neg_marks":"0","meta_id":"3420","dirty":null,"comment":null,"history":"[<b>Feb
05 2021 12:03:43][admin<\/b>][I][<b>admin<\/b>]","answer":"2.00","prefix":null,"content":"In &Delta;ABC if b + c
= 3a , then&nbsp;<img alt=\"cot frac{B}{2}.,cot frac{C}{2}\"
src=\"https:\/\/d2lbh14zkcqlst.cloudfront.net\/content_data\/content_images\/1612331736701856980.png\"
\/>&nbsp;has a value&nbsp;","suffix":null,"testcases":[],"solution":null,"insert_time":"2021-02-05
12:03:43","login_id":"admin","topic_id":0,"hint":null,"qmock_list":null,"unattempt":"2","unattempt_avg_time":"1
7.33","unevaluate":"0","unevaluate_avg_time":"45","correct":"0","correct_avg_time":"45","incorrect":"0","incorr
ect_avg_time":"45","sp_answer":"","mco_marks":"0","exam_ques":"no","exam_info":null,"sub_type":"numerical"
,"is_bonus":"0","is_extra_question":"no","answer_type":null,"integer_length":null,"error_status":"0","source_q_i
d":"1467898","topic":"miscellaneous","subject":"mathematics"}]}}},{"id":"911304","section_id":"45868","sequenc
e_id":"89","sequence_oid":null,"render_gid":"","num_ques":"1","header":null,"content":null,"exam_id":"80","topi
c_id":"0","is_linked":null,"autoHeader":"","qdata":{"89":{"subquestion":[{"id":"993380","group_id":"911304","seq
uence_id":"89","sequence_oid":null,"sequence_sid":"0"

,"render_qid":"89","render_sqid":"","header":null,"type":"subjective","category":"none","is_mco":"no","level":"lo
w","time":"120","marks":"4","neg_marks":"0","meta_id":"3420","dirty":null,"comment":null,"history":"[<b>Feb
05 2021 12:03:44][admin<\/b>][I][<b>admin<\/b>]","answer":"2.00","prefix":null,"content":"The value of
&#39;k&#39; for which equation <img alt=\"{e^{ - |x|}} = k - 1\"
src=\"https:\/\/d2lbh14zkcqlst.cloudfront.net\/content_data\/content_images\/16123317831921086664.png\"
\/>&nbsp;has one solution is&nbsp;","suffix":null,"testcases":[],"solution":null,"insert_time":"2021-02-05
12:03:44","login_id":"admin","topic_id":0,"hint":null,"qmock_list":null,"unattempt":"2","unattempt_avg_time":"1
6","unevaluate":"0","unevaluate_avg_time":"45","correct":"0","correct_avg_time":"45","incorrect":"0","incorrect
_avg_time":"45","sp_answer":"","mco_marks":"0","exam_ques":"no","exam_info":null,"sub_type":"numerical","is
_bonus":"0","is_extra_question":"no","answer_type":null,"integer_length":null,"error_status":"0","source_q_id":"
1467906","topic":"miscellaneous","subject":"mathematics"}]}}},{"id":"911305","section_id":"45868","sequence_id
":"90","sequence_oid":null,"render_gid":"","num_ques":"1","header":null,"content":null,"exam_id":"80","topic_id
":"0","is_linked":null,"autoHeader":"","qdata":{"90":{"subquestion":[{"id":"993381","group_id":"911305","sequen
ce_id":"90","sequence_oid":null,"sequence_sid":"0
","render_qid":"90","render_sqid":"","header":null,"type":"subjective","category":"none","is_mco":"no","level":"
medium","time":"120","marks":"4","neg_marks":"0","meta_id":"3420","dirty":null,"comment":null,"history":"[<b>
Feb 05 2021 12:03:44][admin<\/b>][I][<b>admin<\/b>]","answer":"9.00","prefix":null,"content":"If&nbsp;<img
alt=\"a in left( {0,,,100} right)\"
src=\"https:\/\/d2lbh14zkcqlst.cloudfront.net\/content_data\/content_images\/16123318171143337349.png\"
\/>&nbsp;&nbsp;, then the number of values of a for which the equation [x<sup>2<\/sup>] + x&ndash;a =
0&nbsp;<br \/>\n{where [.] denotes greatest integer function}has atleast one solution
is&nbsp;","suffix":null,"testcases":[],"solution":null,"insert_time":"2021-02-05
12:03:44","login_id":"admin","topic_id":0,"hint":null,"qmock_list":null,"unattempt":"2","unattempt_avg_time":"2
0.33","unevaluate":"0","unevaluate_avg_time":"45","correct":"0","correct_avg_time":"45","incorrect":"0","incorr
ect_avg_time":"45","sp_answer":"","mco_marks":"0","exam_ques":"no","exam_info":null,"sub_type":"numerical"
,"is_bonus":"0","is_extra_question":"no","answer_type":null,"integer_length":null,"error_status":"0","source_q_i
d":"1467915","topic":"miscellaneous","subject":"mathematics"}]}}}]}]}}},"array_ER":[]}

Potrebbero piacerti anche